Federal Courts Outline

Table of Contents

The Judicial Function

The Nature of the Judicial Function (Private v. Public Rights) 1) Where there is a legal right there is a legal remedy a) Marbury v. Madison - Essence of civil liberty exists in right of an individual to claim a remedy to wrongs b) if right is violated, and legal system hasn’t clearly codified what the remedy is, the courts will create one b.i)Marbury v. Madison - the judicial remedy of mandamus is always available (every remedy is a judicial one. If there’s no remedy, they’ll make one up.) 2) It’s emphatically the court’s duty is to say what the law is a) The job of the court is to decide cases and controversies; not to uphold the Constitution, etc. b) However, since it is the court’s job to decide questions under the Constitution, it must “of necessity expound and interpret that rule.” b.i)once it is deciding, the court is authorized to do everything necessary to decide the case (involved passing on Qs of law, interpreting the Constitution, etc.) b.ii) Justifies constitutional review 3) Retroactivity in judicial decision making a) Changes in rules of judicial decision-making must be applied retroactively (Harper v. Virginia Dep’t of Taxation) a.i) Applies to civil and criminal rules b) Selective prospectivity is forbidden b.i)Applying the new rule only to this case but not others pending would be making impermissible choices like a legislature c) Pure prospectivity is forbidden c.i) Applying a new rule only to future cases and not the case of decision and pending cases would defeat point of bringing suit, would be legislature-like 4) The Supreme Court’s Certiorari Power a) “Review of a writ of certiorari is not a matter of right, but of judicial discretion. A petition for a writ of 1 certiorari will be granted only for compelling reasons, including: Court of Appeals split; state supreme court split; state court or Appeals Court has decided important federal question that should be decided by the Supreme Court; and rarely when asserted error consists of erroneous fact findings of application of law.” a.i) More consistent with a “public view”: Court gets to decide if it’s important; this is not deciding on case-by-case basis 5) Constitutional Interpretations Outside the Courts a) Once the Supreme Court rules, contrary interpretations are not allowed (Cooper v. Aaron) 6) Public Rights v. Private Rights views: a) Private rights a.i) The sole function of the federal courts is to decide particular cases about alleged violations of a particular plaintiff’s rights a.ii) Holding statutes unconstitutional, ordering government officials to cease unlawful behavior, and other rulings of significance to society that impact others aside from the parties at bar are incidental to the courts’ case-deciding function b) Public rights b.i)The function of federal courts goes beyond simply deciding particular cases between particular parties b.ii) Federal courts have special responsibilities that aren’t incidental to their case-deciding function, but are themselves justification for judicial action, such as: (b.ii.1) Interpreting and enforcing the Constitution and laws (b.ii.2) Ensuring that federal and state governments behave lawfully 7) SC granting Certiorari: a) Matter of judicial discretion b) Granted only for compelling reasons: b.i)Circuit courts have split on the same matter b.ii) A circuit court has decided an important federal question in a way that conflicts with a decision by a state court of last resort b.iii) A circuit court has so far departed from the usual course of judicial proceedings that the Court should exercise supervisory power b.iv) A state court of last resort has decided an important federal question in a way that conflicts with the decision of another state court of last resort or federal court of appeals b.v) A state court of last resort or federal court of appeals has decided an important question of federal law that hasn’t been (or should be) settled by the Supreme Court, or has decided a case in a way that conflicts with relevant Court decisions c) This reflects a public rights view – the Supreme Court only decides the important cases, as opposed to all cases that could potentially come before it d) SC Rule 46 - If the parties both ask for dismissal, it’s granted (private rights view)

Justiciability

Advisory Opinions 8) Rule: The Courts do not issue advisory opinions (Letter from C.J. Jay to President Washington) a) Exception: federal courts not barred from reviewing state action to determine its constitutionality a.i) SC could review constitutionality of Arkansaws in failing to comply with Brown v. Board (Cooper v.

2 Aaron) 9) Policy (why no advisory opinions?) a) Instrumentalist answers: a.i) Beneficial for the judges to have all the arguments in front of them (a.i.1) (could appt. counsel to argue each side, but doctrinal reasons keep us from doing this) a.ii) “Council of revision’ argument: proposal that after Congress passed a bill, it would go to counsel which could reject the bill, later became the veto power (a.ii.1) It’s rejection is evidence that no one wanted courts to issue advisory opinions (a.ii.2) BUT this may not be a good point; Council of Revision could reject for any reason, whereas the Court must reject for Constitutional reasons only b) Doctrinal answers: b.i)The Constitution creates a case or controversy requirement b.ii) Inherent in the nature of judicial power is not to issue advisory opinions (b.ii.1) However, this was NOT the rule at English courts (M’Naughten’s Case) (b.ii.2) English judges can issue advisory opinions, as can judges in Canada, Germany, ICJ and some US states (MA, FL) c) Other answers: c.i) Three branches intended to be checks on each other c.ii) Court of last resort – may decide questions of law if they come up in cases, but only then. (Private rights view) c.iii) Constitutional grant of power to President to call on heads of departments for opinions seems to be purposely and expressly given to the executive d) NOTE: Declaratory Judgment NOT an advisory opinion: d.i)Declaratory Judgment Act (28 U.S.C. § 2201) d.ii) Willing v. Chicago Auditorium Association (1928) (Brandeis) (d.ii.1) Willing was unsure if his lease permitted him to replace a building it built on its land with a more modern building, and sued for a declaration that he wouldn’t be liable to the Association for doing so (d.ii.2) This case did not present a case or controversy under Article III d.iii) Aetna Life Insurance Co. v. Haworth (1937) (Hughes) (d.iii.1) Affirmed the constitutionality of the Declaratory Judgment Act (d.iii.2) The practice of bringing cases solely for legal or equitable relief in 1789 isn’t the only possible way to present a case or controversy to the courts under Article III d.iv) Dicta does not constitute an advisory opinion (d.iv.1) Monell v. Department of Social Services of the City of New York (1978) (Brennan) (d.iv.1.a) Employees of New York City sought relief under § 1983 on grounds that the city adopted official policies that violated their rights (d.iv.1.b) The Court held that § 1983 permits suit against municipalities, then stated at length that it wouldn’t have permitted the suit if it hadn’t involved an official city policy or established city custom (d.iv.1.c) Stevens, Concurring – the discussion of official city policy and custom was advisory, and not needed to explain the Court’s opinion

Finality and Extrajudicial Functions 10)Rule: Statutes which would render court decisions less than final are invalid (Hayburn’s Case, 1792) 3 a) Actions which are reviewable by executive officials are not really judicial actions a.i) Act allowed veterans to claim pensions by making certain showings to a judge, but made these reviewable by secretary of War, therefore unconstitutional (Hayburn’s case) a.ii) Case which would have required Treasury Secretary to submit an appropriation to effectuate a judgment was not a proper judicial case (Gordon v. United States (1864)) (a.ii.1) No more treasury appropriation, no more problem (United States v. Jones) b) When the judgments of judges are subject to control and review by the legislature they are no longer judicial (separation of powers) 11)Rule: The executive and legislative branches have no power of review over the judicial acts or opinions of a court a) Legislation mandating retroactive review of final decisions violates constitution a.i) Statute retroactively lengthening statute of limitations for securities violations was invalid (Plaut v. Spendthrift Farm, Inc. (1995)) 12)Rule: the federal courts may not act extrajudicially a) Statutes which command the court to take extrajudicial action are invalid b) Exception: Judges may act extrajudicially IF b.i)They are acting in their individual capacity (b.i.1) Distinction: does the statute command a “judge” or the “court” b.ii) A particular extrajudicial assignment wouldn’t undermine the integrity of the court (b.ii.1) Extradition proceedings are fine (Lo Duca v. United States) b.iii) The extrajudicial role is optional – judges may simply volunteer c) Common application: extradition commissioners c.i) §3184 vests judges (individuals) with the power to act extrajudicially 13)NOTE: “Incompatibility clause” of Constitution prevents members of congress from serving in judicial or executive roles a) Clause does NOT apply to judges or executive officials – they can cross over

Standing to Sue 14)Doctrinal Foundations a) Doctrinal arguments: a.i) “Case and Controversy” requirement of Constitution (Allen v. Wright) a.ii) separation of powers (a.ii.1) Courts may not restrict or restrain the actions of congress when not necessary to decide a case (Frothingham v. Mellon) (a.ii.2) Province of legislature to make the laws (a.ii.3) Some matters better left to the political process (Flast v. Cohen) b) Instrumental arguments: b.i)Not requiring standing would “open the floodgates” to too many lawsuits (Frothingham v. Mellon) b.ii) We prefer that the legislature has incentives to get the law right b.iii) Actual injury insures robust argument on each side (b.iii.1) Important for remedy and to ensure no advisory opinions (b.iii.2) Establishment Clause of constitution is important enough to ensure robust litigation (Flast v. Cohen) (b.iii.3) BUT there is no guarantee that those with actual injury will litigate more aggressively or argue more persuasively c) Policy: 4 c.i) Pro-restrictive standing c.ii) Anti-restrictive standing d) Judge Fletcher: standing should go to the merits d.i)Should depend on whether statutory or constitutional claim is raised (d.i.1) Congress has duty to define standing for statutory issues d.ii) Proposed test: (d.ii.1) What law is being violated? (d.ii.1.a) Straightforward – where is the right coming from? (d.ii.2) What is that law trying to achieve? (d.ii.2.a) The incompatability clause gives general rights, therefore anyone should be able to sue on it (Schlesinger v. Reservists) (d.ii.2.b) The right to protect national parks from development is similar to nuisance, should grant users of park legal rights (Sierra Club) e) O’Connor in Allen v. Wright (1984): e.i) Standing embraces judicially self-imposed limits on the exercise of federal judicial power e.ii) Standing requires that a plaintiff allege personal injury fairly traceable to the defendant’s allegedly unlawful conduct and likely to be redressed by the requested relief e.iii) An injury must be distinct and palpable, not abstract, conjectural, or hypothetical e.iv) The standing inquiry requires careful judicial examination of a complaint to ascertain if the particular plaintiff is entitled to adjudication of the claim asserted e.v) The standing inquiry must be answered in reference to: (e.v.1) The Article III notion that federal courts exercise power only as a necessity, and only when adjudication is consistent with a system of separation of powers (e.v.2) Whether the dispute is one traditionally thought to be capable of resolution through the judicial process 15)Procedural notes: a) The standing inquiry focuses on the plaintiff, not the issues sought to be adjudicated (Simon v. Easstern Kentucky Welfare Rights) b) Standing is judged differently for each remedy (and each claim?) see infra 16)Constitutional Injury Requirements: a) Old test: injury requires invasion of legal rights (Tennessee Electric v. TVA) a.i) Rights are those which existed at common law: property, contract, tort, or created by statute b) Modern test (3+ parts): b.i)Injury in fact: (b.i.1) Test: injury must be (b.i.1.a) Concrete and particularized (b.i.1.b) Actual or imminent, not conjectural or hypothetical (b.i.2) “a party must show not only that a statute is invalid, but also that he sustained or is immediately in danger of sustaining a direct injury from its enforcement” (Frothingham) (b.i.2.a) Ideological or political interest is NOT enough (b.i.2.a.i) Disney was granted a bid to construct and operate a ski resort, and a 20-mile road leading to the resort, in the Mineral King Valley in Sequoia National Park, Sierra Club sued for declaratory judgment under the APA as a corporation with a special interest in the conservation of nature, SC says this is not cognizable injury (Sierra Club v. Morton) b.i.2.a.i.1. Dissent (Douglas): inanimate object should have standing (b.i.2.b) Competitive business injury IS enough 5 (b.i.2.b.i) ADPSO, which sells data processing services to businesses, sued Camp, Comptroller of the Currency, to challenge a ruling making data processing services available to banks, SC says this is sufficient (ADPSO v. Camp) (b.i.3) Probability of injury: (b.i.3.a) Test: To have standing, a plaintiff who hasn’t already suffered an injury must show the threatened injury is certainly impending (b.i.3.a.i) Plaintiffs challenged the 2008 amendments to FISA, which authorize the federal government to surveill aliens abroad, on grounds that they communicated with foreigners likely to be targets of such surveillance and their calls would likely be intercepted, SC says too speculative (Clapper v. Amnesty International) (b.i.4) Generalized grievances are NOT injuries (arguably a prudential test) (b.i.4.a) Generalized grievances: (b.i.4.a.i) Taxpayers have standing when they can show sufficient nexus indicating… b.i.4.a.i.1. The status and type of legislative enactment, as opposed to incidental expenditure of funds in administering a law b.i.4.a.i.1.a. Insufficient – allegation that tax exemptions for transferring property to sectarian schools violated the establishment clause was rooted not in congress’s exercise of the tax and spend power but in an agency’s decision to transfer federal property (Valley Forge Christian College v. Americans United for Separation of Church and State) b.i.4.a.i.2. The status and precise nature of the constitutional infringement alleged (i.e., that the challenged law exceeds limitations imposed on the exercise of taxing and spending power, not just that an enactment is generally beyond Congress’s taxing and spending power) b.i.4.a.i.2.a. Insufficient - Taxpayer challenge to the Maternity Act, which provided money to states should they accept and comply with its provisions, challenged under 10th amendment coercion of state sovereignty grounds, was generalized grievance (Frothingham v. Mellon) b.i.4.a.i.2.b. Sufficient - Petitioner taxpayers filed suit to enjoin allegedly unconstitutional expenditure of federal funds to finance instruction in religious schools (Flast v. Cohen) (b.i.4.a.ii) Information-seekers: b.i.4.a.ii.1. Right to know expenditures of the CIA is generalized (US v. Richardson) b.i.4.a.ii.2. FECA requires every PAC to file reports with the FEC listing receipts, expenditures, and donors who have contributed more than $200 per year, confers right to know this info to citizens (FEC v. Akins) (b.i.4.a.iii) Right to restrict congressmen from serving in military (incompatibility clause) is insufficient (Schlesinger v. Reservists Committee to stop the war) (b.i.4.a.iv) alleging that insufficient denial of tax-exempt status to racially discriminatory private schools interferes with their children’s ability to receive an education in public schools was generalized grievance (Allen v. Wright) (b.i.4.b) Exception: Generalized grievances are ok to vindicate fundamental rights (b.i.4.b.i) Interest in Establishment Clause may be sufficient (Flast v. Cohen) (b.i.4.c) PROF: this may make more sense in prudential column (b.i.5) Organizational standing: (b.i.5.a) Organizations must show that some member actually would be injured (Sierra Club v. Morton) (b.i.5.b) Environmental organizations challenged a decision by the U.S. Forest Service to 6 exempt certain land management decisions from allegedly mandatory procedures, alleged that “thousands of members regularly used national forests”, SC denies standing because specific members who will be harmed are not identified (Summers v. Earth Island Institute) b.ii) Causation requirement: (b.ii.1) Rule: the Case or Controversy Clause requires federal plaintiffs to allege a threatened or actual injury that “fairly can be traced” to the challenged action of the defendant, not independent action of an unnamed third party (b.ii.1.a) Government’s granting of tax status to hospital did not cause indigent plaintiffs to be denied care (Simon v. KY Welfare Rights) b.iii) Redressability requirement: (b.iii.1) Rule: plaintiff’s grievance must be likely to be redressed by a favorable decision (b.iii.1.a) Redressability found: (b.iii.1.a.i) Revoking special privilage of women to get SS benefits when not dependant WOULD redress injury of male suing under Equal Protection Clause (Heckler v. Mathews) b.iii.1.a.i.1. Injury was that he was not being equally protected (b.iii.1.b) Redressability not found: (b.iii.1.b.i) Revoking tax exemption of discriminatory private schools would not necessarily lead to change in school policies or change behavior of white parents (Allen v. Wright) (b.iii.1.b.ii) Changing of government’s policy in granting tax-exempt status was too unlikely to cause hospital to accept plaintiffs as patients (Simon v. KY Welfare Rights) (b.iii.1.b.iii) Conviction of father for failing to pay child support would not necessarily lead to him paying it (Linda RS v. Richard D) b.iii.1.b.iii.1. He could just go to jail, ect. (b.iii.1.b.iv) Invalidation of ordinance requiring single family homes would not necessarily lead to indigent plaintiffs acquiring affordable housing (Warth v. Seldin) (b.iii.1.c) Decisions of prosecuting authorities: (b.iii.1.c.i) Citizens lack standing to contest the policies of prosecuting authority when they themselves are neither prosecuted nor threatened with prosecution b.iv) Policy: (b.iv.1) When the government is making a reasonable interpretation of a law, the courts are hesitant to upset it (b.iv.1.a) Particularly when the interpretation might not be cause of plaintiff’s injury (b.iv.2) Courts don’t want to take away traditional prosecutorial discretion of the government b.v) NOTE: re-characterizing injury can completely change standing (b.v.1) If Heckler was asking for money, he wouldn’t have had a case 17)Prudential standing requirements: a) Rule: parties can’t litigate another person’s legal rights a.i) Exception: Third Party Standing: (a.i.1) Third parties generally do not have standing to sue for those who have an actual injury (a.i.1.a) Maryland citizens did not have standing to sue under First Amendment for defendants who were criminally prosecuted for selling alcohol on Sundays (McGowan v. Maryland) (a.i.2) Requirements for Third Party Standing (3-part test): (a.i.2.a) Relationship of litigant to the person whose rights are being asserted (a.i.2.a.i) Vendor-vendee (alcohol vendor) relationship may be sufficient (Craig v. Boren) (a.i.2.b) Ability of person to advance his own rights 7 (a.i.2.b.i) When affected party is not party to racial covenant, party to contract may enforce their rights (Barrows v. Jackson) (a.i.2.c) Impact of the litigation on 3P interests (a.i.2.d) Other guarantees of effective advocacy (a.i.2.d.i) When contract will be enforced against the plaintiff, she may bring action on behalf of racial minority (Craig v. Boren) (a.i.3) NOTE: third party must still have some injury from the challenged actions, just need not be as severe as standing normally requires (a.i.4) No standing to sue for violations of Constitutional rights by private parties (a.i.4.a) However, may have standing to sue government officials even if the statute could potentially be applied to private parties (US v. Raines) (a.i.4.b) The threat of government enforcement of an unconstitutional private contract is enough to confer standing (Craig v. Boren) (a.i.5) Special rule conferring standing in First Amendment Claims (a.i.5.a) Because unconstitutional speech statutes might discourage speech and thus prevent an actual injury for a challenge, there is no need to show personal injury (a.i.5.b) Threat that statute banning “opprobrious” language would chill future speech was enough to grant standing to defendant whos speech was not constitutionally protected (Gooding v. Wilson) (a.i.6) Third party standing situations: (a.i.6.a) Law is unconstitutional as applied, but only because it violated 3P’s rights (Craig) – OK! (a.i.6.b) (2A) Law is constitutional as applied, but could violate constitutional rights in some other cases (Raines) – NOT OK! (a.i.6.c) (2B) Law is constitutional as applied, but could violate constitutional rights in some other cases – and is about Free Speech (Gooding) – OK! b) Rule: plaintiff must be within law’s “Zone of Interests”: b.i)“Zone of interest” comes from §702 of the APA (Clarke v. Securities Industry Ass’n) (b.i.1) Whenever federal gov’t does anything, you are entitled to judicial review as long as you are adversely affected or aggrieved (injury) within the meaning of the statute (zone of interest) b.ii) Congress must have intended to protect class plaintiff is within b.iii) Two-part test: (b.iii.1) What interests was congress protecting? (b.iii.1.a) Goal of statute establishing Postal Service monopoly was to protect postal revenues (Air Courier Conference v. APWU) (b.iii.1.b) Goal of Federal Credit Union Act was to limit the market of the credit union (National Credit Union v. First National Bank) (b.iii.2) Does plaintiff possess those interests? (b.iii.2.a) Statute defining Postal Service’s monopoly on mail service was not intended to protect the interests of postal workers themselves (Air Courier Conference v. APWU) (b.iii.2.b) Sharing the “interests” of the intended group may be enough (b.iii.2.b.i) Commercial banks’ shared interest in limiting the markets of Federal Credit Unions is sufficient for “zone” test (National Credit Union v. First National Bank) b.iv) Reconciling the Court’s holdings on “zone of interests” (b.iv.1) Cynical theory: (b.iv.1.a) Business interests usually have standing (b.iv.1.b) Advocacy organizations (liberal ones) usually do not 8 (b.iv.2) Principled theory: (b.iv.2.a) Plaintiff must be more than an “incidental” beneficiary (b.iv.2.a.i) Businesses have a financial interest that is concrete and easy to understand b.iv.2.a.i.1. In Haven Realty, advocacy group made its living supporting racial plaintiffs (b.iv.2.a.ii) Advocacy organizations have less concrete interest, would be too hard to know where to draw the line 18)Congressional Control of Standing and Remedies: a) Rule: congress may ONLY dispense with prudential standing requirements a.i) Constitutional standing requirements are constitutionally mandated a.ii) Congress can’t instruct court to hear case when plaintiff isn’t injured b) Rule: congress can create standing by passing laws creating legal rights b.i)Congress must explicitly identify what the injury is in the statute to confer standing by statute (b.i.1) Sufficiently explicit: (b.i.1.a) Stating that anyone has standing to enforce right to PAC information is sufficient (FEC v. Atkins) (b.i.1.b) Right to all “persons” to truthful information about available housing is sufficient for tester who is the object of misrepresentation (Havens Realty Corp. v. Coleman) (b.i.2) Insufficiently explicit: (b.i.2.a) Requiring a specific administrative process does not create generalized standing to see that it is adhered to (Lujan v. Defenders of Wildlife) c) Question: is there an outer limit on Congress’s ability to statutorily grant standing? c.i) Lujan majority – Yes (c.i.1) Congress cannot confer rights when it would be a “generalized grievance” (c.i.2) There should be no distinction between constitutional and statutory claims (Scalia dissent in Atkins) c.ii) Atkins majority – No (c.ii.1) Congress can always confer standing if they do it with sufficient specificity 19)Particular Remedies: a) Standing judged differently for each remedy sought a.i) Forward-looking remedies (injunction) require proof of forward-looking injury (a.i.1) Risk of injury is generally not an injury in itself (Public Citizen) (a.i.2) Plaintiff is unlikely to suffer another illegal chokehold by police department, therefore no standing to sue for injunction (City of LA v. Lyons) (a.i.2.a) To get an injunction, Lyons had to show he would (a.i.2.a.i) have another encounter with the officers, (a.i.2.a.ii) that all officers always place citizens they encounter in chokeholds, or (a.i.2.a.iii) that Los Angeles ordered or authorized this behavior (a.i.3) civil penalties may be available because of their deterrent effect (a.i.3.a) Friends sued Laidlaw for discharging mercury into the North Tyger River in amounts exceeding the levels fixed in its permit issued under the Clean Water Act, SC allows suit because civil fines may deter pollution (Friends of the Earth, Inc. v. Laidlaw Environmental Services) a.ii) Fines paid to the government have deterrence value which may grant plaintiffs standing (a.ii.1) Fine paid to US gov’t for environmental pollution was sufficient for plaintiffs complaining of aesthetic and recreational injury (Friends of the Earth v. Laidlaw) 9 Mootness 20)Plaintiffs must have standing throughout the time a case is pending a) “standing in a time frame” b) “collateral consequences” may be sufficient b.i)even though the criminal conviction being appealed has been served, the collateral consequences which attach to a criminal conviction (can’t vote or get a gun permit, subject to deportation, etc.) make the conviction sufficient (Sibron v. NY) c) Class action certification is sufficient c.i) Typically, claim must not be moot when class certification occurs (c.i.1) Class action appropriate for one-year residency requirement prior to seeking divorce (Sosna v. Iowa) c.ii) If certification prior to mootness is impractical, may not be required (c.ii.1) Plaintiffs claiming illegal pretrial detention could have standing because certification during short detention period was impractical (Ferstein v. Pugh) c.iii) Class action not moot as long as it is live for some class plaintiffs (even if not for named plaintiff) (c.iii.1) Named plaintiff in employment discrimination claim fired for other legitimate reasons did not moot case (Franks v. Bowman Transport Co.) c.iv) Mooting the substantive claims does not moot a class certification (c.iv.1) Even though plaintiff’s class was not certified, court would look to mootness of class’s claims, not just the plaintiff, when deciding whether to hear class certification appeal (US Parole Comm’n v. Geraghty) 21)Exceptions to the mootness requirement a) “Capable of Repetition, Yes Evading Review” a.i) Challenged action is too short in duration to be fully litigated prior to its cessation, and (a.i.1) pregnancy, ect. a.ii) Reasonable expectation that the same complaining party would be subjected to the same action again (a.ii.1) unlikelihood that parole violee would be subject to parole violation again defeated standing (Weinstein v. Bradford) (a.ii.2) applied with varying stringency (Roe, ect.) b) Voluntary cessation b.i)If defendant voluntarily ceases activity but reserves legal right to start again, case is not moot (b.i.1) Polluter’s right to restart polluting made case against him not moot (Friends of the Earth v. Laidlow) 22)Policy: a) Good: a.i) Protects separation of powers – limits court to its constitutional role a.ii) Gives courts discretion to pass on politically difficult questions b) Bad: b.i)Doctrine doesn’t seem to be applied with any consistency b.ii) Lack of clarity of the language in Art. III? b.iii) Restraint from justiciability doesn’t necessarily save us from judicial activism

10 Political Question 23)Real v. Bogus political question a) Bogus Political Question: a.i) Failure to state a claim for which relief can be granted (a.i.1) Federal judge does not have standing to sue for lack of proper trial by the House of Representatives that led to his impeachment (Nixon v. US) b) Real political question: b.i)A particular decision is vested to another branch of government (b.i.1) Court had no standing to hear suit by congressman against decision by House of Representatives to give election to his opponent because decision constitutionally given to the House itself (Morgan v. US) (b.i.1.a) House must be “elected by the people” – constitutional constraint 24)Six Factors for determining Political Questions (Baker v. Carr) a) Textually demonstrable commitment to another political department a.i) Commitment comes from the constitution (a.i.1) The power to judge the outcome of elections is textually committed to the House of Representatives (Morgan v. US) a.ii) Even if a decision is committed to another branch of government, the courts may judge the extent of that commitment and whether it was based on the required criteria (a.ii.1) Court may judge whether House is acting within it’s committed responsibilities when seating its members (Powell v. McCormack) b) Lack of judicially manageable standards b.i)Standards or text the Court is asked to decipher are not judicially manageable (b.i.1) Note: courts often interpret ambiguous language b.ii) Courts are capable of determining whether state electoral districts violate the Equal Protection Clause (Baker v. Carr) c) Requires initial policy determination of a non-judicial character c.i) Courts should not decide whether to recognize a particular country (Taiwan, ect.) d) Requires expressing a lack of respect for another government branch d.i)Don’t want to overrule another branch unless they have to? (mysterious) e) Unusual need for adherence to a political decision e.i) Typically invoked in the foreign policy context e.ii) For the good of the nation-type argument – Rhode Island f) Potential for embarrassment of the government f.i) Again, most prevalent with foreign policy f.ii)No embarrassment in cases of constitutional interpretation

Congressional Control of Jurisdiction

Congress’s Power over Jurisdiction 25)Article III, § 2 – “In all Cases affecting Ambassadors, other public Ministers and Consuls, and those in which a State shall be Party, the supreme Court shall have original Jurisdiction. In all the other Cases before mentioned, the supreme Court shall have appellate Jurisdiction, both as to Law and Fact, with such Exceptions, 11 and under such Regulations as the Congress shall make.” 26)Traditional doctrine (plenary power over federal jurisdiction) a) Control of lower courts: Congress has plenary power over the lower federal courts because it has the authority to create or destroy them a.i) The jurisdiction of the lower courts extends no farther than congress has authorized (a.i.1) Even though a mortgage bond case technically complied with diversity requirements, congress has not authorized that kind of diversity jurisdiction (Sheldon v. Still) a.ii) Does NOT extend as far as constitutionally permissible a.iii) Congress’s creation of the federal courts did not automatically mean they intended to vest them with jurisdiction b) Control of Supreme Court: Congress has plenary power over the Supreme Court under the “exceptions” clause b.i)May make any exceptions to SC jurisdiction that it deems necessary b.ii) Includes exceptions to the appellate power under the exceptions clause (b.ii.1) Congress may remove Habeas Corpus jurisdiction in order to affect the outcome of pending libel case (Ex Parte McCardle) b.iii) Exception: congress does not control the Supreme Court “original” jurisdiction (b.iii.1) This is quite limited, see constitutional text above 27)Constraints outside Article III (“external constraints”) a) Rule: Congress may not limit jurisdiction by mandating a rule of decision or specific outcome a.i) “Congress can’t require a court to deny itself jurisdiction previously conferred because its decision, in accordance with settled law, is adverse to the government” (a.i.1) Congress could not nullify the pardon of former confederate in order to deny him property that would otherwise be his (US v. Klein) (a.i.2) law requiring agencies to permit timber sales subject to certain restrictions, and affecting pending cases by environmental groups, didn’t establish a rule of decision – it changed the law and directed courts to apply the new law to pending cases, which is permissible (Robertson v. Seattle Audubon Society) b) Rule: Congress may not remove jurisdiction when it would impair executive authority b.i)Statute that made grant of presidential pardon sufficient to prove disloyalty (can’t claim pre-civil war property) was invalid for effectively limiting president’s pardon power (US v. Klein) b.ii) Bumedian v. Birk(Bush) – violating suspension of Habeas Corpus clause is enough b.iii) Congress may not discriminate in grants of jurisdiction in violation of the Equal Protection clause c) Rule: Congress can’t exercise its power to give, withhold, and restrict lower federal court jurisdiction in a way that would deprive people of due process or take their property without just compensation c.i) Portal-to-Portal Act retroactively eliminated liability for underpaying workers, and provided that no court would have jurisdiction over any action to enforce the liabilities, Act didn’t deprive employees of property because rights under the FLSA are subject to Congress’s power to regulate commerce, even retroactively (Battaglia v. General Motors Corp. (2d Cir. 1948)) d) Rule: Congress may not violate an explicit constitutional mandate d.i)USCS Const. Art. I, § 9, Cl 2. “Suspension clause”: “The Privilege of the Writ of Habeas Corpus shall not be suspended, unless when in Cases of Rebellion or Invasion the public Safety may require it” d.ii) In 2006, Congress passed a law stripping state and federal courts of jurisdiction over applications for habeas relief by aliens accused of being enemy combatants, This law violates the Suspension Clause because congress didn’t authorize sufficient alternative (Boumediene v. Bush) e) Theory: congress may not take away jurisdiction in a way that would fundamentally burden a constitutional right e.i) Example: it may be unconstitutional for congress to strip courts of all jurisdiction of First Amendment 12 claims 28)Constraints inside Article III (“internal constraints”) a) NOTE: THIS STUFF IS BASICALLY ALL THEORY!!! b) Rule: no one really knows what the hell’s going on here c) What we do know: c.i) Congress retains plenary power over Supreme Court appellate jurisdiction c.ii) Congress retains plenary power to deny original jurisdiction in the “last six” categories mentioned in Article III (c.ii.1) Controversies to which the United States shall be a Party; (c.ii.2) Controversies between two or more States; (c.ii.3) between a State and Citizens of another State; (c.ii.4) between Citizens of different States; (c.ii.5) between Citizens of the same State claiming Lands under Grants of different States, and (c.ii.6) between a State, or the Citizens thereof, and foreign States, Citizens or Subjects d) What we’re not sure about: d.i)Congress must vest the federal courts with jurisdiction over the “first three” categories of article III, but may decide where to vest it (Story/Amar theory) (d.i.1) Cases arising under federal law (d.i.2) Cases involving ambassadors (d.i.3) Cases in admiralty and maritime e) Options available to congress for vesting jurisdiction: e.i) Jurisdiction choices of congress: (e.i.1) original/appellate, (e.i.2) mandatory/discretionary, (e.i.3) lower/supreme f) Rule: congress cannot remove jurisdiction of SC to rule on the constitutionality of a law? g) Hart’s View of Article III Limits on Congress’s Power Over Jurisdiction g.i) Congress has plenary power to limited federal court jurisdiction when the consequence is merely to force proceedings to be brought in state court g.ii) If federal rights are at stake, state courts may be obligated to vindicate them g.iii) Congress can impose exceptions to federal court jurisdiction so long as it doesn’t destroy the essential role of the Supreme Court under the Constitution g.iv) Benefits: (g.iv.1) Protects the courts and the Constitution, consistent with early congressional practices, consistent with the Exceptions Clause g.v) Drawbacks: (g.v.1) Some textual problems, this is a balancing test so it’s unclear how far is too far h) Black’s View of Article III Limits on Congress’s Power Over Jurisdiction h.i)Congress can impose any exception to the Supreme Court’s appellate jurisdiction h.ii) The Court can’t be given the right to make exceptions to its own jurisdiction if Congress, which was expressly given this power, can’t exercise this right h.iii) Federal courts take jurisdiction when told to, and don’t when told not to h.iv) Benefits: (h.iv.1) Clear, easy to apply, gives the judiciary democratic legitimacy, respects the Exceptions Clause

13 h.v) Drawbacks: (h.v.1) Contradicts some constitutional text, could take away constitutional rights i) Wechsler’s View of Article III Limits on Congress’s Power Over Jurisdiction i.i) Congress’s ability to limit jurisdiction is unlimited i.ii)Federal courts pass on constitutional questions because they must decide issues in their jurisdiction, not because they have a special function to enforce the Constitution or police other governmental entities i.iii) Still, Congress can’t completely take away jurisdiction over certain cases j) Story/Amar’s View of Article III Limits on Congress’s Power Over Jurisdiction: j.i) Comes from Justice Story, Martin v. Hunter’s Lessee (1816) (j.i.1) Supreme Court of Virginia refused to implement SC judgment saying they had no appellate jurisdiction j.ii)Congress didn’t have to create inferior federal courts j.iii) The first 3 categories of federal judicial power are distinct from the remaining 6 (j.iii.1) Judicial power of the U.S. is vested in the federal judiciary, not the state judiciary (j.iii.2) Judicial power must, at minimum, grant subject matter jurisdiction to finally decide all cases in the first 3 categories (j.iii.3) Judicial power may, but need not, extend to cases in the other 6 categories j.iv) Congress can shift final resolution of any case in the Supreme Court’s appellate jurisdiction to any other Article III court that Congress may create j.v) Article III affirms the parity of all federal judges (j.v.1) Appointment, confirmation, tenure, and salary are the same for all judges (j.v.2) This is why lower courts can be trusted with finally resolving cases in the first 3 categories, and state courts can’t unless subject to federal review j.vi) Criticism: this doesn’t account for the early constitutional practice keeping the Supreme Court from reviewing state statutes struck down for violating federal law or the Constitution j.vii) Application of theory: (j.vii.1) Stripping lower courts of jurisdiction over certain federal question is fine (j.vii.2) Allowing only lower courts to hear a certain federal question is fine (j.vii.3) Not allowing any federal court to hear an abortion challenge is not fine j.viii) Benefits: (j.viii.1) Respects “all” and “shall”, we like the federal courts can enforce our rights, fairly clear j.ix) Drawbacks: (j.ix.1) Ignores parts of Article III, a bit intricate, inconsistent with the 1st Judiciary Act k) Policy: k.i)It is advantageous that we don’t know the answer to how much power congress has over Supreme Court’s jurisdiction (k.i.1) Congress could take this power too far if it could (k.i.2) The Court could assume too much power if it couldn’t 29)Constraints on “channeling” jurisdiction: a) Rule: congress has broad power to channel jurisdiction where it pleases b) Definition of jurisdiction channeling b.i)Permissible channeling: “no court can hear a case about this statute” (b.i.1) not as problematic, because a constitutionality defense is not precluded b.ii) impermissible channeling: “when a court hears a case, it can’t consider the constitutionality of the statute”: (b.ii.1) This deprives a forum for hearing the law, precludes a defense (more serious than precluding a claim) 14 c) Rule: As long as congress grants plaintiffs the ability to be heard in some federal tribunal, the courts have no jurisdiction to hear the plaintiff’s claims c.i) Congress’s grant of jurisdiction to a special court to hear challenges to the Emergency Price Control act was sufficient, courts could not hear plaintiff’s case absent grant of jurisdiction from congress (Yakus v. US) d) Interpretation of jurisdiction-stripping statutes: d.i)Courts require “clear and convincing evidence” that congress wanted to restrict access to judicial review (d.i.1) Eliminating all challenges “arising under” a statute completely strips jurisdiction and is unconstitutional d.ii) Congress will normally view a statute as jurisdiction “channeling” (d.ii.1) Veternan conscientious obector sued VA which had denied Robison’s application for educational assistance, SC says the “no review” clause does not restrict constitutional review, no indication that the VA was ruling on that question, constitutional review not explicitly prohibited (Johnson v. Robison) (d.ii.2) Statute giving completely unfettered authority to CIA director to hire and fire did not preclude constitutional claims because congress’s intent was not clear (Webster v. Doe)

Control of State Jurisdiction 30)History: a) Federalist No. 82 - States had the pre-existing authority to hear these claims, and the creation of the federal courts didn’t divest them of these claims 31)Rule: States are presumed competent to hear cases under federal law a) Basis: State courts are usually of general JD, which typically is empowered to hear cases arising under any law (even foreign) a.i) So if it can hear a case arising under law of Japan, it can certainly hear a federal law case a.ii) State courts have concurrent jurisdiction over RICO claims (Tafflin v. Levitt) b) Exception: Congress may divest a state court of concurrent jurisdiction, ONLY when it is explicit (or possibly when very clearly implying it) c) Exclusivity is established via congressional intent c.i) Indications of exclusively (Tafflin v. Levitt factors) (c.i.1) Expressly stated in statute (express) (c.i.2) Legislative history (implied) (c.i.3) Compatibility of federal and state jurisdiction (implied) c.ii) NOTE: the SC seems to overrule implied exclusivity in Yellow Freight Systems (c.ii.1) Lack of express language in Title VII divesting state court of jurisdiction under EEOC appears to have been dispositive, though legislative history was considered 32)Removal from courts: a) Normally, cases removable to federal court only when complying with the “well-pleaded complaint rule” (see “federal jurisdiction” infra) a.i) to determine if complaint arises under federal law, only look at the complaint, NOT the defense a.ii) can remove from state to federal only if the case could have started in federal court originally b) Exception: §1442: Removal is permitted in criminal prosecutions for alleged violations of state law where defenses are claimed under federal law or authority (see federal jurisdiction, infra) b.i)For constitutional purposes, a case “arises” under federal law if there’s a Fed Q anywhere in it

15 b.ii) Davis was indicted for murdering Haynes in Tennessee state court and prayed for removal on the ground that he killed Haynes in self-defense in carrying out his duties as a federal revenue agent in attempting to seize an illegal distillery, SC says case removable to federal court (TN v. Davis) b.iii) Constitutional basis: probably necessary and proper clause c) Where federal court takes jurisdiction on removal, state court proceedings must cease (TN v. Davis) 33)Rule: state courts must hear federal claims unless they have a valid, non-discriminatory reason not to a) State courts cannot decline to hear federal causes of action a.i) Claiming a federal statute is against state policy is not an adequate ground for failing to hear the federal cause of action (Mondou v. State of NY) a.ii) Federal governmet is not a “foreign sovereign” and states must enforce federal penal statutes (Testa v. Katt) b) Exception: state courts may decline to hear federal actions when it has a “valid excuse” b.i)Test: Basically states need an excuse beyond the fact that it’s a federal law, some policy that would apply to all laws b.ii) Valid excuse found: (b.ii.1) Where plaintiff, defendant and facts of case are all from out of state, state may have a valid excuse for not hearing the case (Douglas v. NY) (b.ii.2) Missouri state courts refused to dismiss a federal case brought by a non-resident against an out-of-state corporation for an accident occurring outside of Missouri because they didn’t think they could invoke forum non conveniens to dismiss a federal cause of action, SC says this fine because was local policy which did not discriminate against federal claims (Missouri ex rel. Southern Railway Co. v. Mayfield) c) NOTE: Thomas in Haywood v. Drown (2009) says history shows framers didn’t think state courts were required to hear federal claims 34)State court procedure when hearing federal claims: a) State procedure controls unless would defeat an “integral part” of the federal right a.i) Allowing for a non-unanimous jury verdict does not defeat federal rights (Minneapolis & St. L.R. Co. v. Bombolis) a.ii) Allowing judge to hear factual issues regarding fraud in a FELA case does defeat an integral part of the the federal right, because the right to trial by jury of all issues of fact is too substantial a part of the rights under FELA to not be observed by state courts (Dice v. Akron, Canton, and Youngstown R. Co.) a.iii) Simplified federal pleading rules are integral to a federal right (Brown v. Western Ry. Of Alabama) b) Policy: b.i)Siegel: Frankfurter position makes more sense, because it’s confusing trying to work with “hybrid” model, and P always has another choice (bringing in federal court) (b.i.1) But if Congress says something (“must have unanimous jury verdict”), seems like state courts would have to apply this b.ii) Alternative argument: As long as there is a fair procedure, they should just apply that to federal law (b.ii.1) But this isn’t the rule – look to “integral part of process” 35)Policy: a) Why can Congress may make federal jurisdiction exclusive? a.i) Authority: (a.i.1) Supremacy clause (a.i.1.a) PROF: technically, this clause doesn’t confer any specific powers (a.i.2) Necessary and proper clause (a.i.2.a) Congress can make federal JD exclusive if it’s the best vehicle to carry on a certain power

16 a.ii) BUT can’t abridge pre-existing rights (a.ii.1) I.e, could take away diversity JD from states b/c didn’t exist at the time of the union b)

The Applicable Law

§ 34 of the Judiciary Act: “The laws of the several states, except where the Constitution, treaties, or statutes of the U.S. otherwise require or provide, shall be regarded as rules of decision in trials at common law, in the courts of the U.S., in cases where they apply.” State Law in Federal Courts 36)Procedural principle – Federal law is interstitial: a) can partially govern a situation, but leave other matters to state law, a.i) leads interpretation of federal law to vary from state to state even though the cause itself is universal b) It’s a matter of statutory interpretation whether federal law is interstitial b.i)In determining how much state law is displaced by federal law, courts should evaluate if the objective of the statute would be frustrated by the application of varying state law 37)Federal interpretation of state law: a) Old doctrine (Swift v. Tyson) a.i) Federal courts apply state written law, but federal common law b) Erie doctrine (Erie RR Co. v. Tompkins): b.i)Rule: In diversity cases, federal courts apply the common law of the states in state causes of action b.ii) There is no general federal common law – federal courts aren’t empowered to create it (b.ii.1) Rules of Decision Act: “laws of several states shall be the laws of decision when state law apply” (b.ii.2) Practical effect: discriminates against instate Ps – doesn’t fulfill proper purpose of diversity JD (b.ii.3) Constitution: No clause in Constitution gives power to Congress to control issues of state law b.iii) Paradigm shift in theory of what law is - it’s something that emanates from a sovereign gov’t b.iv) Exception: Congress has paramount power to fix and determine national maritime law (b.iv.1) Federal maritime law applies in case about death of maritime worker (Southern Pacific Co. v. Jensen) c) Policy: c.i) Problems with the Swift doctrine: (c.i.1) State court common law differed by state, preventing uniformity of decision (c.i.2) It was difficult to demarcate general law from local law (c.i.3) Swift led to forum-shopping to get more favorable results (c.i.4) Many people were moving out-of-state to establish diversity (c.i.5) There was uniformity among federal courts, but not among federal and state courts sitting in the same jurisdiction 38)Choice of law principles: 17 a) Federal court, sitting in diversity, must apply the choice of law principles of the state in which it is sitting (Klaxon v. Stentor Electric MFG Co., Inc.) a.i) Policy: Don’t’ want different results in each court – also encourages forum shopping b) Test: When does state law apply? b.i)When there are state law claims and federal law claims in the same case (b.i.1) Exception: doesn’t apply to federal question cases where state law is applicable by virtue of a federal statute, if the statute specifies a different rule (b.i.1.a) IE, in FTCA case, statute says that proper law is that of the place where the claim arose, and this controls even if this is different than where the court is b.ii) When federal statute refers back to state law b.iii) When federal statute leaves out reference to which law applies c) Change of venue rules: c.i) following a change of venue at the defendant’s request, the transferee court should apply the same law that would have applied in the first court (Van Dusen v. Barrack) c.ii) this also applies to cases where the plaintiff requests a change of venue after first filing a case in a proper forum (Ferens v. John Deere, Co.) d) Policy: d.i)Two options for uniformity (d.i.1) Vertical: More certainty with federal law principles in state and fed courts (d.i.1.a) Erie thought it was more important to have vertical uniformity (d.i.1.b) Simply the result of the federal system where each state has its own law (disuniformity among states is a result of this system) (d.i.2) Horizontal: More certainty between federal courts in different states (d.i.2.a) Under Swift, horizontal uniformity was promoted because the thought federal judges would come to some sort of general federal law d.ii) What does current policy of using CoL principles of state where federal court sits accomplish? (d.ii.1) Enforces vertical uniformity (between courts in same state), but horizontal disuniformity between same courts in different states (d.ii.1.a) Policy perspectives: forum shopping not so great (d.ii.2) Siegel: under Erie regime, is choice of law a state or federal matter? (d.ii.2.a) Swift: choice was a “general” law matter (d.ii.2.b) Many different approaches to CoL: (d.ii.2.b.i) Better law approach: Courts decide which law is better, and apply that one (d.ii.2.b.ii) First Rest. (common): turned on where the event occurred (d.ii.2.c) Interest Analysis: (d.ii.2.c.i) How state law interests would be affected by certain law (d.ii.2.d) MSR: most significant relationship (d.ii.3) If CoL is an extension of substantive law, then state laws should apply in the absence of Congressional override 39)Statute of Limitations principles: a) Rule: Normally, where federal statute creates a cause of action, but doesn’t put a SoL on that action, court should find analogous state law (state where court sits?), and apply that SoL a.i) Rule in almost all cases b) Exception: federal court may draw SoL from a federal statute when necessary based upon three factors (Agency Holding Corporation v. Malley-Duff & Associates): b.i)There is no analogous state law (b.i.1) No state law is analogous to RICO (Agency Holding Corporation) 18 b.ii) Uniform SoL is necessary (b.ii.1) RICO actions are interstate, go across the country, need uniformity b.iii) Finds Congressional intent (b.iii.1) This has been the practice for many years, Congress is familiar with it, and they would expect this result (b.iii.2) Intent may be inferred when a statute is modeled on a previous federal statute (RICO via Clayton Act) c) Contrary policy argument – Scalia concurrence: c.i) Notion of “borrowing” state law is inappropriate - state SoL applies of its own force (c.i.1) Until federal law preempts, the state law applies (they’re general gov’ts) c.ii) Federal law can displace all of state law in certain areas; can also just displace some of state law in a certain area (“interstitial” quality) (c.ii.1) I.e., if federal law creates only a cause of action, then only that preempts state law, but all other aspects of the case are governed by state law (c.ii.2) This isn’t disuniformity, because the federal law (cause of action), remains the same throughout all states d) NOTE: In 1990, Congress enacted a catchall 4-year federal statute of limitations for federal causes of action not subject to specified statutes of limitations, in laws enacted after 1990 40)“Traditional province of state law” principles: a) When federal law requires a legal determination which is typically the province of state law, court will look to the state where (the case is tried?) to make the determination a.i) Federal court determining definition of “child” in copyright case applied law of California (DeSylva v. Ballentine) b) Factors favoring looking to state law: b.i)Congress did not contemplate the issue when it passed the relevant statute (b.i.1) Congress was not contemplating family law when passed copyright protections b.ii) Only one state is involved c) Policy: c.i) The scope of a federal right is a federal question, but that doesn’t mean it isn’t to be determined under state law, particularly when a law pertains to familial relationships created under state law c.ii) Contrary view (Douglas) – uniformity is better for everyone

Federal Common Law 41)Federal common law applies where there is a sufficient federal interest at stake 42)Areas in which federal common law applies a) Cases involving the rights and duties of the United States b) Cases involving Other Federal Interests c) Instructions to create federal common law c.i) Express: rare (c.i.1) FRE 501 (criminal law privileges) c.ii) Implied (c.ii.1) Congressional grant of jurisdiction over arbitration agreements in labor disputes implied authority to create federal CL (Textile Workers Union of America v. Lincoln Mills) (c.ii.2) Critical point in figuring out whether JD grant implies grant to use FCL is the purpose in the grant of JD 19 (c.ii.2.a) Was the grant of jurisdiction created in order to establish a uniform, neutral forum? c.iii) Statutory interpretation: bears a strong resemblance to the creation of federal CL, especially when a statute is so vague that its interpretation necessarily entails a great deal of judicial discretion c.iv) Kimball foods (c.iv.1) Who has the best claim on a lein (c.iv.2) Court says that this 43)Rule: Federal common law applies when… a) The rights and duties of the US are at issue a.i) IE the US is a party to the case a.ii) Federal law governs questions of the U.S.’s rights under federal programs (a.ii.1) Priority of liens from SBA federal lending programs must be evaluated under federal law (United States v. Kimbell Foods) a.iii) When federal law is unclear, federal courts to fill in the interstices of the federal legislation based upon the following factors: (a.iii.1) Federal programs requiring uniformity necessitate controlling federal rules (a.iii.1.a) Issuing of commercial paper occurs on a vast scale in several states and requires uniformity (Clearfield Trust Co. v. United States) (a.iii.2) If application of state law will frustrate specific objectives of federal programs, need to fashion special rules solicitous of those objectives (a.iii.2.a) Rights and duties of the United States under its commercial paper are sufficient (a.iii.3) Evaluate the extent to which applying the federal rule would disrupt commercial relationships or private expectations predicated on state law b) Other federal interests are at stake – courts should consider… b.i)Whether the case involves a uniquely federal interest (b.i.1) Uniquely federal interests: (b.i.1.a) Obligations to and rights of the U.S. under its contracts (Boyle v. United Technologies Corp.) (b.i.1.b) Civil liability of federal officials for actions taken in the course of their duty (Boyle) b.ii) Whether there is a significant conflict between federal and state law (b.ii.1) Test: displacement of state law happens only where a significant conflict exists between an identifiable federal policy or interest and an application of state law

c) Congress expressly authorizes the creation of federal common law (rare) c.i) FRE 501 states that except as otherwise required by the Constitution, federal law, or the Supreme Court’s rules, privilege is governed by common law as interpreted by federal courts. (c.i.1) However, in civil actions where state law supplies the rule of decision, privilege is governed by state law d) Congress implies authorization to create common law d.i)Labor Management Relations Act § 301(b) makes it possible for a labor organization sue and be sued in federal court, while § 301(a) supplies the basis for a court to take jurisdiction and apply § 301(b) (d.i.1) Court finds this is sufficient invitation to fasion common law from federal labor law (Textile Workers Union of America v. Lincoln Mills of Alabama) 44)Policy: a) Why shouldn’t federal courts be free to apply common law? a.i) Courts can’t be held accountable the way that legislatures can a.ii) Under separation of powers principles, the judiciary shouldn’t have the power to make laws outside 20 of the boundaries set in D’oench a.iii) The federal government is limited, so the starting point must be state law unless it’s been displaced under the Supremacy Clause a.iv) Unclear when statutory interpretation stops and federal common law begins 45)Kimball foods: a) Case regarding priority of leins on SBA small business lending program after bankruptcy b) Incorporating state law here would in no way hinder the SBA’s administration because… b.i)SBA employees are instructed to follow state law, and they do so b.ii) SBA Manual indicates security interests are controlled by state commercial law b.iii) Local lending offices and employees are familiar with local law b.iv) Choosing responsible debtors requires individualized selection procedures, which the SBA has already implemented in great detail b.v) Applying state law won’t frustrate the SBA’s program objectives b.vi) There is less need here than in the tax lien area (where the object is speedy tax collection) to invoke protective measures against defaulting debtors

Rights of Action 46)Rule: Where there is a legal right there is a legal remedy 47)Classic approach: a) If plaintiff is within the class of persons meant to be benefitted, they have a right of action a.i) RR employee was injured on railroad car at work by a defective handle. Employee could sue because Federal Safety Appliance acts provided for safety standards, including proper handles on cars, meant to cover the plaintiff (Texas & Pacific Ry. Co. v. Rigsby) 48)Mid-century approach: a) Where the authorities cannot be expected to reasonably enforce a federal law, the courts may fashion a right of action a.i) SEC was overwhelmed with securities fraud duties, therefore shareholders were not protected as congress intended (J.I Case Co. v. Borak) b) Four factor test to determine whether plaintiff is within the protected class (Cort v. Ash): b.i)Is P within class of intended beneficiaries? (b.i.1) Shareholder was not intended beneficiary of law forbidding corporations from spending money in federal elections (Cort v. Ash) b.ii) Indication of legislative intent? (b.ii.1) Only look to see if Congress clearly intended that there wouldn’t be a private right of action b.iii) Consistent with underlying purposes of the legislative scheme? b.iv) Is cause of action one traditionally falling under state law? (b.iv.1) Corporate law is the law of the states, not federal (Cort v. Ash) 49)Current approach: a) Test: there is no private right of action unless congress affirmatively intended to create one a.i) Cort factors are simply evidence of congressional intent (a.i.1) However, focus on congressional intent doesn’t mean evidence is required that Congress, in enacting the statute, actually had creation of a private cause of action in mind (Thompson v. Thompson) (a.i.2) Prof: this makes no sense since congressional intent is one of the four factors 21 a.ii) Inquiry may be limited to the text of the statute itself (a.ii.1) Statutory language required filing the reports, and both the statute and legislative history were silent on the question of remedies, so this is dispositive of a private right of action under §17(a) of Securities Act (Touche Ross v. Reddington) b) Alexander v. Sandoval – § 601 of the Civil Rights Act provides that no one may be discriminated against by a program receiving federal financial aid § 602 – directs federal agencies to effectuate § 601 by setting rules, regulations, or orders b.i)SC finds the rights-creating language in § 601 is completely absent from § 602, which only describes the rights and duties of federal departments and agencies b.ii) Creation of agency-centered method of enforcement is evidence congress did not intend for § 602 to be enforced by private parties c) NOTE: recent court opinions have suggested that congressional intent is the primary factor, or perhaps the only factor c.i) (Cannon v. Univ. of Chicago) c.ii) (Touche Ross) c.iii) (Thompson v. Thompson) 50)Policy: a) Potential approaches a.i) Rigsby: if P was meant to be benefited, there’s a cause of action a.ii) Cort: 4 factors: Intended beneficiary, legislative intent, purpose of scheme, traditionally falling under state law a.iii) Cannon: 4 factors of intent (a.iii.1) Focus on one factor, but take into account others (a.iii.2) Intent is only factor (a.iii.3) No factors b) Consequences of various approaches: (1) Size of the penalty, (2) Type of enforcement b.i)Centralized (gov’t): only one thing empowered to bring actions, so there will be much fewer actions (b.i.1) But maybe we only want the egregious cases brought b.ii) Decentralized (private parties): more likely that suits will be brought (b.ii.1) But maybe we don’t want every single action being brought c) Is requiring explicit statutory text to create right of action a good idea? c.i) Idea that this is a legislative function, and that the judiciary can’t make this call c.ii) Courts’ free-formed interpretations of legislative intent may be wrong c.iii) Courts should encourage the passage of good legislation c.iv) Sometimes the government doesn’t enforce laws it disagrees with d) Congress could resolve this issue by implementing a default provision regarding private causes of actions any time it wants 51)Express private rights of action: a) Where suit is against a state official: 42 USC §1983: whenever someone under color of state law deprives any citizen of any right, privileges or immunities guaranteed under the Constitution shall be liable in an action at law (expressly, there is a right of action) b) Where P in suit against federal official seeks damages other than money damages: 5 USC §702: express right of action

22 Federal Jurisdiction

Judicial Power Under Article III 52)Rule: Federal courts can only hear cases that fall within one of the nine enumerated Article III bases for jurisdiction a) Under Article III, a case arises under federal law if there is any federal question somewhere in the case, even if the federal question is ultimately never raised b) The possibility that a defendant will raise a federal question is enough to grant the federal courts jurisdiction under the “arising under” jurisdiction b.i)OH tried to levy a tax on a federal bank in OH; although contract arose under state law, an issue of federal grant of authority to bank might have arisen, therefore federal court had jurisdiction (Osborn v. Bank of the US (US 1824)) (b.i.1) PROF: this is clearly wrong – too broad c) Cases arising under the Foreign Sovereign Immunity Act arise under federal law c.i) A foreign corporation sued an entity of the Nigerian government in federal court for breach of the Foreign Sovereign Immunities Act, SC says that even if the main rule of decision would be derived from state law, the case necessarily involves the threshold federal question of whether the defendant was immune to suit (Verlinden B.V. v. Central Bank of Nigeria (1983)) d) A congressional charter’s ‘sue and be sued’ provision may be read to confer federal court jurisdiction if it specifically mentions suit in federal court d.i)The Red Cross was entitled to remove a tort action against it to federal court (American National Red Cross v. S.G. (1992)) 53)Protective jurisdiction: a) Congress appears to have the authority to simply put a case under state law in federal court by explicit command (“protective jurisdiction”) a.i) § 301 implicitly instructs federal courts to create federal common law governing collective bargaining agreements, the suit was one that ‘arose under’ federal law (Textile Workers Union of America v. Lincoln Mills of Alabama) 54)Removal jurisdiction: a) If case start with Fed Q, but comes up later: could get to federal court by removal b) Jurisdiction over Federal officials: b.i)Claims against officers of the United States may be removed to federal court under § 1442 only when they claim a federal defense (b.i.1) § 1442 authorizes the removal of any civil or criminal action brought against any officer of the United States for any action committed under the color of his office (b.i.2) California charged Mesa, a federal postal worker, with manslaughter after she accidentally killed a bicyclist with her postal truck, didn’t raise federal defense so no removal (Mesa v. California) 55)NOTE: If Article III didn’t cover questions containing both federal and state law issues, there could be no appeal to the Supreme Court from state supreme courts

Statutory Jurisdiction 56)28 USC § 1331 - Federal question – “The district courts shall have original jurisdiction of all civil actions arising under the Constitution, laws, or treaties of the United States.”

23 a) ‘Arising under’ is less expansive in § 1331 than it is in Article III a.i) Interest in keeping out cases where defendants don’t raise federal defenses a.ii) Federal courts can’t hear purely state law cases between interstate parties a.iii) Would deprive state courts of cases they have sole authority to hear a.iv) Federal courts are courts of limited jurisdiction a.v) A more expansive interpretation is included in the Constitution to facilitate Supreme Court review b) A case can ‘arise under’ federal law in 2 ways: b.i)When federal law creates the cause of action asserted (majority of cases) b.ii) When a state law claim raises a federal issue within the meaning of Grable 57)28 USC § 1441 - Removal of civil actions – “Except as otherwise expressly provided by Act of Congress, any civil action brought in a State court of which the district courts of the United States have original jurisdiction, may be removed by the defendant or the defendants, to the district court of the United States for the district and division embracing the place where such action is pending.” a) permits only defendants to remove cases, and only cases where federal courts would have had original jurisdiction b) Accordingly, under the well-pleaded complaint rule, defendants may only remove if the plaintiff could have brought the case in federal court in the first place 58)Declaratory Judgment Act – states that in a case of actual controversy, any federal court can declare the rights and legal relations of any party seeking such declaration, whether or not further relief is or could be sought 59)The Well-pleaded complaint rule: a) Rule: The ‘arising under’ clause only confers jurisdiction when the plaintiff’s statement of his own cause of action shows it is based on federal law or the Constitution a.i) In establishing federal court jurisdiction, plaintiffs must rely only on their claims b) cases may be brought originally in federal court when the plaintiff alleges a federal cause of action (Louisville & Nashville R. Co. v. Mottley) c) “arising under” in the context of a federal statute is interpreted much more narrowly than “arising under” in Article III c.i) contrast with Osborn 60)State law incorporating federal law: a) Old rule: jurisdiction depends only upon if state or federal law gives the cause of action a.i) Suit for slander that requires determination of the validity of a federal patent is a suit under state law (American Well Works Co, v. Layne & Bowler Co.) a.ii) The fact that state law incorporates federal law is insufficient for federal jurisdiction (a.ii.1) Kentucky statute determining liability by reference to the Federal Safety Appliance Act did not raise a federal question (Moore v. Chesapeake and OH Ry. Co.) a.iii) Exception: it may be good enough if a cause of action requires P to establish a point of federal law (a.iii.1) P s/h sued company seeking to enjoin company from purchasing illegal bonds (claim was breach of fiduciary duty, but case would require determining whether federal bond program was constitutional, was sufficiently under federal law (Smith v. Kansas City Title & Trust Co.) b) New rule: a state law raises a federal question ONLY IF there is a private right of action under the federal statute b.i)Suit brought by mothers who had taken drug, resulting in birth defects, claiming that Bendectin violated the FDCA – say the drug was misbranded under the statute. Violation of the safety statute was a rebuttable presumption of N. Court holds no jurisdiction because FDCA doesn’t create a statutory right of action (Merrell Dow Pharmaceuticals v. Thompson) c) New New rule: federal courts have jurisdiction over a state law claim which raises a stated federal issue, disputed and substantial, which a federal forum may hear without disturbing congressionally approved

24 balance of federal and state judicial responsibilities c.i) IRS seizes property from G, D buys the property, G brings suit against D because land was improperly seized, because whether Grable was given notice in compliance with federal law is an essential element of its claim, and the meaning of the law is actually in dispute, federal court had jurisdiction (Grable & Sons Metal Products, Inc. v. Darue Engineering & Manufacturing (2005)) c.ii) Relevant considerations: (c.ii.1) Clear legislative interest present? (c.ii.2) Extent to which the JD would intrude on state issues (c.ii.3) Emphasis on the # of cases that would be coming into federal court (c.ii.3.a) Should this be a consideration? (c.ii.3.a.i) No: we either have JD or we don’t (c.ii.3.a.ii) Yes: where JD is not so clear, this is an appropriate consideration to maintain the balance between state and federal courts (c.ii.4) Congressional judgment (c.ii.4.a) 1875 “arising under statute” or §1331 statute: Congressional judgment is only one sentence – this seems more like a judicial judgment c.iii) Four-part test for determining jurisdiction over state law claim: (c.iii.1) necessarily raised; (c.iii.2) actually disputed; (c.iii.3) substantial; and (c.iii.3.a) forward-looking claims are much more likely to be substantial (c.iii.3.a.i) malpractice suit against attorney under patent case was backward-looking, wouldn’t affect future patent cases (Gunn v. Minton) (c.iii.4) capable of resolution in federal court without disrupting the federal/state court balance approved by Congress (c.iii.4.a) states have primary responsibility for professional lawyer standards (Gunn v. Minton) (c.iii.4.a.i) no evidence of congressional intent to override this

Supplemental Jurisdiction 61)Rule: Federal courts may exercise jurisdiction over all claims when there is at least one federal claim and all claims arise under the same nucleus of operative facts a) When all federal claims are dropped from a case, federal court loses jurisdiction (United Mine Workers of America v. Gibbs) b) § 1983, suit for violation of constitutional rights by government officials, impliedly forbids court from exercising pendent JD (Aldinger v. Howard) c) § 1332 suit in diversity forbids the impleading of a non-diverse defendant (Owen v. Kroeger) d) (Finley v. US) e) Distinguish from Osborn e.i) Osborn had one claim wrapped up in both federal law and state law (in that case the federal court must resolve all of these claims) e.ii) Here, these are 2 different claims, each distinctly federal and state (court must try the federal claim only) 62)Congress finally passes §1367, which gives a codified basis for pendent JD (now called “supplemental JD”) a) Reflects congress’s interpretation of “cases,” in the Constitution (this is constitutional) 25 b) In any civil action where federal courts have original jurisdiction, they have supplemental jurisdiction over all other claims so related to claims in the action that they form part of the same case or controversy under Article III, including claims involving joinder or the intervention of additional parties c) A district court may decline to exercise supplemental jurisdiction if: c.i) The claim raises a novel or complex issue of state law c.ii) The claim substantially predominates over the claim or claims over which the district court has original jurisdiction c.iii) The district court dismissed all claims for which it had original jurisdiction c.iv) In exceptional circumstances, there are other compelling reasons to do so d) § 1367 unambiguously provides jurisdiction over the claims of unnamed members of a class (Exxon Mobil Corp. v. Allapattah Services (2005))

Lawsuits Against Governments

Suits Against the Federal Government 63)Rule: The federal government can’t be sued, unless consent is given by Congress a) The government’s waiver of immunity must be clearly stated in the statutory text (Lane v. Pena) a.i) Legislative history is irrelevant b) US government was only liable for direct damages caused by the Indian tribes (Price v. US) 64)Old Rule for avoiding FSI: a) Agents of the government may be sued under legal fiction that they are private citizens (Marbury v. Madison) a.i) If the rights of the plaintiff are of a kind normally sought against individuals, court will entertain the action (a.i.1) Grandson of Robert E. Lee sues to eject federal government from his land (US v. Lee) b) If the plaintiff is seeking money the court is less likely to entertain the action (Little v. Barreme) b.i)This is because the court knows that the government will ultimately be paying the judgment against the official c) There is no immunity for officials acting illegally or outside the scope of statutory authority, even if the public official acts on executive orders c.i) Statute that authorized seizing ships going to france did not grant immunity for seizing ships traveling from France, regardless of the fact that president had authorized the latter (Little v. Barreme) c.ii) Plaintiff may stop the government seizure of the steel mills during Korean War was permissible because secretary of state was acting beyond the scope of his constitutional authority (Youngstown Steel Seizure case) c.iii) A good faith belief that one’s orders were lawful is no defense (c.iii.1) Bates’s plea that he had good reason to believe the seizure occurred in “Indian country” and acted in good faith is no defense to this action (Bates v. Clark) 65)Mid-century adjustments to FSI: a) Court may dismiss the suit if it’s not directed at the right official a.i) Suit against liquor commissioner during prohibition was dismissed because was not against commissioner of internal revenue (Gnerich v. Rutter) a.ii) Court allows suit to require Maritime Commission to return stock to Dollar Steamship company 26 (Land v. Dollar) (a.ii.1) Concurrence: this makes no sense, the officials would have to act in their official capacity to grant relief, destroys the legal fiction b) Two types of cases where officers can be sued: b.i)Officer being sued in personal capacity (b.i.1) Officer is acting as sovereign if actions would be attributable to government under normal laws of agency (b.i.1.a) Larson was statutorily authorized to enter into the contract and refuse delivery of coal (Larson v. Domestic & Foreign Commerce Corp) (b.i.2) If relief would require officer to take official action, fails this test (b.i.2.a) Court ignores this requirement in Land v. Dollar, requires officer to officially transfer property from the government commission’s possession to the plaintiffs b.ii) Officer being sued where actions are beyond statutory limitations/pursuant to unconstitutional statute (b.ii.1) b.iii) Court rejects third category: where officer taking actions pursuant to an unlawful order c) Suits will be dismissed if they would either (Land v. Dollar)… c.i) “expend itself on the public treasury or domain,” (cost mad $$$) OR c.ii) interfere with public administration 66)Current rule for abrogating FSI: a) APA (U.S.C. § 702): Person wronged with a cause of action shall not be dismissed on the grounds that the US is an indispensable party or is named in suit, provided that the names of the officers you’re suing are given b) Relief available: b.i)Monetary relief not available under the APA b.ii) Congress has waived FSI for monetary damages in two instances: (b.ii.1) Tucker Act: waives immunity from K claims (b.ii.2) Federal Tort Claims Act: waives immunity for tort claims (b.ii.2.a) Very important exceptions to this act (b.ii.2.b) Boyle v. United Technologies – implemented the “discretionary function” exception to the FTCA b.iii) Federal government is never liable for consequential damages even when it has allowed itself to be sued c) NOTE: suing a public official is still available, but not much incentive to use it 67)Policy: a) Theories of sovereign immunity a.i) Holmesian Theory: (a.i.1) There can be no right against the authority that makes the law upon which the right depends (gov’t makes the law that gives you rights) (a.i.1.a) If gov’t defines torts as acts committed by private parties, this isn’t a tort. Nothing gov’t does can be a tort a.ii) Hamilton Theory (a.ii.1) Simply inherent in the nature of sovereignty a.iii) Arguments: Natural v. positive law – go against and for (a.iii.1) Ks with sovereign are only binding on conscience of the sovereign a.iv) Public Policy: 27 (a.iv.1) Government can’t be stopped in its tracks by any P who has a dispute (a.iv.2) Courts shouldn’t interfere with duties of legislature (nothing but mischief!) a.v) Mistake/ History (Jaffe) (a.v.1) We inherited much of our law from England, and though they had the maxim “the king can do no wrong,” we misinterpreted this (a.v.1.a) There was always redress provided when the king did a wrong (in the end) a.vi) Mystery theory (a.vi.1) No one knows why we have this immunity – Supreme Court has never given one (a.vi.2) S. Ct. CA calls it a “mystery of legal evolution” a.vii) Constitutional thoery (a.vii.1) Scalia: appropriations clause embodies the principle of sovereign immunity (a.vii.2) Siegel: dubious b) Differences between governments and corporations which justify different treatment b.i)Similarities: (b.i.1) Everyone should respect rights (b.i.2) Both act through agents, fictional entities (b.i.3) Injury and justice can be the same for both b.ii) Differences: (b.ii.1) Choice in corps, no choice in gov’t (b.ii.2) Gov’t public goals, corp. private goals (b.ii.3) Elect gov’t leaders, can’t vote out corp. (b.ii.4) Gov’t creates rights (b.ii.5) Different goals – profit v. non-profit entities

Suits Against States 68)11th Amendment, 1795 a) Judicial power of the US shall not be construed to extend to any suit in law or equity, commenced or prosecuted against one of the United States by citizens of another state, or by citizens or subjects of any foreign state. a.i) Passed in reaction to SC case saying states sueable (Chisholm v. Georgia): 69)Rule: States are subject to federal law, but generally enjoy a presumption of sovereign immunity from suits for not complying a) States waive immunity only when waiver is stated unequivocally by express language or an overwhelming implication in statutory language a.i) Acceptance of federal funds isn’t a waiver of sovereign immunity (a.i.1) California protected from citizen suit alleging discrimination under the Rehabilitation Act (Atascadero State Hospital v. Scanlon) b) Highway department of Texas immune from suit by former employee (Welch v. Texas Department of Highways and Public Transportation) c) Applies to in-state and out of state citizens (Hans v. Louisiana) c.i) Fears the “anomalous result” of construing the 11th Amendment literally to bar only suits by out-of- staters d) States cannot be sued by foreign states, either (Monaco v. Mississippi (1934)) 70)Methods for avoiding SSI: a) Rule: State officials may be enjoined from illegal actions under federal law even where suit against state would not be entertained 28 a.i) State officials lose sovereign immunity protection when they act illegally under federal law (Ex Parte Young) a.ii) This includes where officer has mandatory (ministerial) task but not where he has discretion a.iii) There is NO requirement that the cause of action take a common law action form (a.iii.1) Note: this contrasts with Lee, methods for getting around federal sovereign immunity b) Exception: state retains SSI from federal law claim if they are a “real party in interest” b.i)Test: whether relief sought against the nominal defendant would operate against the state, especially by imposing liability damages to be paid from the public fisc (b.i.1) This applies even if the act was performed in the course of official duties (b.i.2) Sovereign immunity only applies if the judgment must, under all circumstances, be paid out of state funds (b.i.2.a) A plaintiff claimed that UGA employees infringed his copyright, seeking damages. District Court allows suit (National Association of Boards of Pharmacy v. Board of Regents of the University System of Georgia (M.D. Georgia 2008)) b.ii) State IS real party in interest: (b.ii.1) Cases where monetary relief is sought (b.ii.1.a) This includes claims for retroactive relief (b.ii.1.b) Suit to retroactively award wrongfully withheld benefits was barred by SSI (Edelman v. Jordan) (b.ii.1.c) Exception: monetary relief may be sought from state officials depriving plaintiffs of federal rights under state law (Hafer v. Melo) (b.ii.2) Cases which are the “functional equivalent of a quiet title action” (b.ii.2.a) Indian tribe may not sue a state and state officials claiming ownership of submerged lands (Idaho v. Couer d’Alene Tribe of Idaho) b.iii) State is NOT real party in interest: (b.iii.1) Discharged state employees sued the official who fired them under § 1983, alleging they were fired on the basis of their political affiliations, and sought damages, SC says case can go through (Hafer v. Melo) (b.iii.2) State official is denying federal rights to others under color of state law (b.iii.3) Cost of complying with court orders does not change this equation (see “relief available”) c) Relief available: c.i) Rule: courts can grant only give prospective injunctive relief (c.i.1) D can be ordered to follow the federal law in the future, but no retroactive damages – (c.i.2) Distinguishes between suits brought in equity and law (c.i.2.a) unclear where they get this distinction – 11th Amendment includes both (c.i.3) Monetary relief more directly affects the state’s treasury c.ii) Exception: relief which is ancillary to prospective relief is allowed (c.ii.1) Injunctions against state officials can be enforced by jail time or fines (Hutto v. Finney) (c.ii.2) Cost of complying with a court order is ancillary to judgment (c.ii.2.a) Suit against Arkansas correctional officials, claiming the conditions in the Arkansas prisons violated the 8th Amendment, could be enforced and costs imposed (Hutto v. Finney) (c.ii.3) costs and attorneys fees are ok d) Counties and municipal governments have no sovereign immunity (Lincoln County v. Luning) 71)Suits Against States in State Courts: a) States have absolute sovereign immunity under their own laws 29 a.i) Federal courts have no power to grant relief for claims arising under state law (a.i.1) Pennsylvania citizens with mental disabilities challenged their confinement in state institutions under the 8th and 14th Amendments, seeking state to conform its prisons according to state law, SC says this is a state law claim and has no power to grant relief under 11th amendment (Pennhurst State School & Hospital v. Halderman) a.ii) States have sovereign immunity wholly independent of 11th amendment and actions of US congress (a.ii.1) SSI not expanded or limited by the 11th Amendment a.iii) State doesn’t implicitly consent to suit simply by engaging in otherwise lawful activities that are regulated by federal statutes (a.iii.1) probation officers sued Maine in federal court for violating the overtime provisions of the FLSA, seeking monetary and liquidated damages, SC says this is barred by SSI (Alden v. Maine) 72)Current method for abrogating SSI: § 1983 a) Where suit is against a state official: 42 USC § 1983: whenever someone under color of state law deprives any citizen of any right, privileges or immunities guaranteed under the Constitution shall be liable in an action at law (expressly, there is a right of action) 73)Congressional abrogation of SSI: a) Test: to abrogate SSI congress must… a.i) unequivocally express intent to abrogate sovereign immunity (a.i.1) must be clear from the text of the statute? (a.i.2) The ADA validly abrogates state sovereign immunity insofar as it prohibits exclusion from public benefits or services on the basis of disability (Tennessee v. Lane) (a.i.3) Congress validly abrogated state sovereign immunity in the provision of the FMLA requiring employers to provide 12 weeks of unpaid leave to care for sick family members (gender discrimination) (Nevada Department of Human Resources v. Hibbs) a.ii) act pursuant to a valid exercise of power (a.ii.1) Congress may abrogate SSI under its 14th Amendment powers (a.ii.1.a) Based on § 5 of 14th Amendment (“The Congress shall have power to enforce, by appropriate legislation, the provisions of this article”) (a.ii.1.b) Plaintiffs may get prospective monetary relief in title VII suit (employment discrimination) against a state (Fitzpatrick v. Bitzer) (a.ii.2) Congress may NOT abrogate SSI under any constitutional power which predated the 11th Amendment (a.ii.2.a) Congress may NOT abrogate SSI under its Article I powers (a.ii.2.a.i) Seminole sued Florida and its governor under IGRA for failure to negotiate in good faith in creation of casino, SC says state immune from suit in federal court (Seminole Tribe of Florida v. Florida) (a.ii.2.b) Congress may not abrogate SSI under commerce clause powers (a.ii.2.b.i) Seminole overruled Pennsylvania v. Union Gas Co. (federal agency allowed to implead PA under a commerce clause statute) b) Final test: To use enforcement powers against states, congress must: b.i)Identify conduct transgressing 14th amendment (b.i.1) Age discrimination not enough because subject to only rational basis review under 14th amendment b.ii) Tailor its legislative scheme to remedying or preventing such conduct (b.ii.1) Congress passed a law expressly abrogating sovereign immunity for patent-related claims, but this was too far out of proportion to remedial goal of preventing infringment by states (Florida Prepaid Postsecondary Education Expense Board v. College Savings Bank) c) In Rem jurisdiction: 30 c.i) When the state acts as a creditor, it is not immune under in rem jurisdiction (c.i.1) Trustee in bankruptcy is allowed to undo transaction that bookstore had gone into against the state (Central VA Community College v. Katz) c.ii) Congress’s power to pass uniform bankruptcy law permits Congress to empower courts to issue ancillary orders enforcing in rem jurisdiction c.iii) In ratifying the Constitution, the states acquiesced in subordination of sovereign immunity to effectuate the in rem jurisdiction of bankruptcy courts c.iv) There is a special need for uniformity in bankruptcy law 74)Policy: a) Four possible theories of 11th Amendment: a.i) Official theory – states immune from suit unless waived (and ex parte Young exception) (a.i.1) Good – long precedent, protects states (a.i.2) Bad – cuts against disjunctive theory of Article II, enforcement problems, textual problems a.ii) Diversity theory – states have no pre-existing SI, can be sued in diversity like anyone else (a.ii.1) Good – federal courts can enforce federal law, historically sound, textually plausible (a.ii.2) Bad – leads to radical results, doesn’t account for word “any” a.iii) “Literal” theory – do exactly what text says, suit can be brought against state by its own citizens, not by others (a.iii.1) good – respects the text (a.iii.2) bad – enforcement problem, leads to anomalous results a.iv) Akhil Amar Theory (not really 11th Amend.) – states are not really sovereigns at all (a.iv.1) In voting to ratify constitution, people of each state voted to give up their sovereignty and merge into sovereign US as a whole (a.iv.1.a) Can change constitution against the wishes of a particular state (see Art. V) (a.iv.2) States have control over their own law, but suing under federal law means that state no longer controls – if the “whole people” of US creates a cause of action under federal law in which states are suable, then they are (a.iv.3) For the federal gov’t: Should be a distinction between Constitutional claims and other claims (a.iv.3.a) Congress passing statute is like states making state law - If under patent law, federal gov’t is not suable, that’s fine – but this wouldn’t apply if suing under constitutional law (there, the “sovereign” has spoken) b) How are states like and unlike the federal government for sovereign immunity purposes? b.i)Similar (b.i.1) State and federal governments are politically directed, not-for-profit, and democratic (b.i.2) States can’t be sued on the basis of laws they create, unless they consent b.ii) Different (b.ii.1) If a state is sued in federal court, and judgment is entered against it, that judgment can be enforced by the federal government (b.ii.2) Explicit constitutional text provides for federal sovereign immunity, while Article III, § 2 provides that a state can be sued c) Which side does history fall on? c.i) Brennan: History doesn’t support the idea that the Framers intended to constitutionalize state sovereign immunity (c.i.1) Article III reveals that states are capable of being defendants (c.i.2) Constitutional Convention records don’t show substantial controversy over the clauses 31 concerning categories of federal cases (c.i.3) Mason and Madison suggested limits to state sovereign immunity during the Virginia debates (c.i.4) Madison, Hamilton, and Marshall were referring to suit under the Contract Clause for debts incurred during war, not sovereign immunity in general (c.i.5) Framers’ repeated references to the need for federal courts to exercise extensive jurisdiction suggests that Congress could provide rights of actions of states, and federal courts could hear them (c.i.6) Justices Jay and Wilson drafted Article III, and found in Chisholm that a state could be sued in federal court, and Justice Iredell didn’t argue that a state could never be sued in federal court (c.i.7) A proposal for a broader interpretation of state sovereign immunity in the text of the 11th Amendment was rejected d) Why can congress only abrogate SSI with powers enacted after 11th Amendment? d.i)Temporal rationale: since 14th amendment was passed after the 11th, it’s understood to limit it. Since Art. I existed before, 11th modifies that/ overrules to some extent that part of Art. I d.ii) PROF: This reasoning seems attenuated – no one thinks the 14th amendment sweeps away the 8th amendment

Abstention

The Anti-Injunction Act 75)Anti-Injunction Act [28 USC §2283]: A court of the United States may not grant an injunction to stay proceedings in a State court except… a) as expressly authorized by Act of Congress, or a.i) Test: congress must have created a specific and uniquely federal right or remedy, enforceable in federal court, that could be frustrated if the court wasn’t empowered to enjoin a state proceeding a.ii) The court recognizes implicit authority granted by congress (a.ii.1) History of § 1983, combined with vague language suggesting state court should halt actions, were enough to find congressional intent for injunctive authority (Mitchum v. Foster) (a.ii.1.a) § 1983 is a uniquely federal remedy meant to be used against states b) where necessary in aid of its jurisdiction, or b.i)protecting federal rights is NOT the same as protecting federal jurisdiction (Atlantic Coast Line RR Co.) c) to protect or effectuate its judgments. c.i) A federal decision not to enjoin an activity does NOT justify overruling the injunction imposed by a state court on picketing labor protestors (Atlantic Coast Line RR Co.) 76)Interpretation of the AIA: a) “The specifically delineated exceptions in the Act are to be interpreted strictly” (Atlantic Coast Line RR Co.) b) “The Anti-Injunction Act isn’t a flexible doctrine of comity – it imposes an absolute ban on federal injunctions in state proceedings unless an exception applies” (Mitchum v. Foster) c) State proceedings should ordinarily proceed without federal court intervention, with any relief from error to come from state appeals courts and the Supreme Court 32 d) 77)Federal JD is presumed to be concurrent – exclusive only where Congress says it is so a) Rule: The federal government will not enjoin a parallel proceeding in state court (see “congressional control of jurisdiction” supra) a.i) Parallel actions for contract liability does not require federal courts to exercise exclusive jurisdiction (Kline v. Burke Construction Co.) b) Exception: cases brought under in-rem jurisdiction b.i)Reasoning: If jurisdiction is based on a piece of property, two courts cannot have conflicting judgments over the same physical piece of property c) Exception: relitigation of issues decided in federal court c.i) Conversely, once a state court rules on an issue, the Full Faith and Credit Act takes priority over this exception d) Exception: When the plaintiff is the U.S. or a federal agency and it asserts superior federal interests e) Exception: federal injunctions may be granted in exceptional circumstances such as: e.i) Where irreparable injury is both great and immediate e.ii) Where state law flagrantly and patently violates express constitutional provisions e.iii) Where there’s a showing of bad faith, harassment, or other unusual circumstances calling for equitable relief f) Exception: Federal courts can enjoin state court proceedings that interfere with federal class actions 78)Practical considerations for choosing federal/state court: a) Location of the courthouse to lawyer’s office (amount spent is affected) b) Different discovery rules (lots of tactical maneuvering that goes on in trying to get a case in a specific court) 79)Policy: a) Why was the Anti-Injunction Act passed? a.i) States retained powers that weren’t surrendered in ratifying the Constitution a.ii) Our 2 essentially separate legal systems couldn’t function if state and federal courts were free to fight for control over particular cases b) If the state court gets it wrong, can always appeal to state appellate or to SC b.i)State courts still play primary role in enforcing federal rights

Pullman Abstention 80)"the federal courts should not adjudicate the constitutionality of state enactments fairly open to interpretation until the state courts have been afforded a reasonable opportunity to pass on them." a) State court should be allowed to rule on discriminatory action by state Railroad Commission requiring that no sleeping cars be operated on any line of any railroad in Texas unless they are continuously in charge of conductors (who were white) (Railroad Commission of Texas v. Pullman Co.) 81)Rule: Pullman abstention is allowed when there is an unsettled question of state law that, if resolved, could avoid the need for constitutional adjudication. a) Three necessary conditions for invoking Pullman: a.i) The case presents both state grounds and federal constitutional grounds for relief; (a.i.1) A hard question of state law, without a constitutional issue, is insufficient to allow a federal court to dismiss a case back to state court (Lehman Brothers v. Schein) a.ii) The proper resolution of the state ground for the decision is unclear; and a.iii) The disposition of the state ground could obviate the need for adjudication of the federal 33 constitutional ground. (a.iii.1) When state and federal law question are inexorably intertwined, no abstention b) Exception: for action at law (for money), no Pullman abstention (Quackenbush v. Allstate Insurance Co.) c) Exception: if a state constitutional issue is same to federal constitutional provision, no abstention necessary (Quackenbush v. Allstate Insurance Co.) 82)Procedural issues: a) A plaintiff bringing a suit in state court must inform the state court of the federal issues in the case so the state court can decide questions of state law in light of any federal constitutional issues that may arise a.i) A party may reserve his right to return to district court for disposition of his federal claims if state courts hold against him on state law, so long as he invokes Windsor a.ii) A litigant isn’t denied the right to return to district court unless it’s clear he voluntarily did more than Windsor requires and fully litigated his federal claims in state court (Government Employees v. Windsor) b) Once the state court has ruled on a claim, Pullman no longer applies and any federal court may hear it b.i)in federal court for an injunction and declaration that a Louisiana law was unconstitutional as applied, and the district court abstained in accordance with Pullman, England sued in state court and lost on both the state law and constitutional claim, Rather than seeking Supreme Court review, England returned to federal district court to attempt to argue his federal constitutional claim, SC says federal court must now hear it (England v. Louisiana State Board of Medical Examiners (1964)) 83)Certification procedure: a) Can make a reservation in state court to the resolution of your federal claim by the state court, then you can definitely go back to federal court after the state law issue has been resolved (England v. Louisiana State Board of Medical Examiners) a.i) even if you don't make such a reservation, you can usually go back to fed. court b) Certification procedure has effectively replaced the use of Pullman abstention. 84)If state and federal constitutions raise the same issue, courts should not abstain 85)Policy: a) core reasons for abstention: appropriate for fed court to abstain where case presents difficult Q of state law which could avoid the necessity of deciding the federal question a.i) If decided, can go back to federal court for deciding the question b)

Younger Abstention 86)federal courts may not hear civil rights tort claims brought by a person who is currently being prosecuted for a matter arising from that claim in state court 87)Rule: no federal court is to issue an injunction or issue a declaratory judgment against a pending state criminal proceeding a) Always applies to criminal prosecutions? a.i) Jehovah’s Witnesses wanted to go door-to-door in Jeanette, Pennsylvania, and the city adopted an ordinance requiring all solicitors to obtain a license and pay a tax, Douglas was arrested, prosecuted, and threatened with further arrests and prosecutions. SC says court must abstain because no threat of injury beyond normal criminal proceedings (Douglas v. City of Jeannette) b) Applies to certain civil proceedings where there is significant state interests b.i)civil nuisance case against owners of porn theater (Huffman v. Pursue, Ltd.) b.ii) State has an interest in enforcing the orders and judgments of its courts after plaintiffs sought federal injunction of state court judgment against oil company (Pennzoil Co. v. Texaco, Inc.) (b.ii.1) NOTE: Texaco was prejudiced by its failure to raise federal constitutional issues during 34 state trial so federal court didn’t know if it should take jurisdiction b.iii) Nuisance proceeding against the owner of an adult movie theater was sufficiently criminal in nature (Huffman v. Pursue, Ltd.) b.iv) Child custody cases? b.v) NOTE: possibly applies to all state proceedings now 88)Younger Exceptions (no abstention): a) Law flagrantly and patently violates constitutional provisions a.i) First Amendment claims are subject to a lower bar because of the chilling effect of these laws (a.i.1) The plaintiffs sued the Louisiana governor and other state officials to enjoin prosecution on grounds that state law facially violated the 1st Amendment Free Speech Clause, SC does NOT abstain even though defendant probably guilty in this case (Dombrowski v. Pfister) b) Brought in bad faith by the prosecutor c) Cases where state prosecution is threatened but not actually pending c.i) Petitioners distributed handbills protesting involvement in the Vietnam War outside a shopping center, and personnel asked them to stop and leave, Police were summoned and told the group they would be arrested if they didn’t leave, 2 days later, Steffel and a companion resumed handbilling, and the police intervened, Steffel left to avoid arrest, but his companion remained and was arrested, SC says threat but no prosecution meant feds could hear the claim (Steffel v. Thompson) c.ii) Three previous free speech prosecutions over the course of five weeks were sufficient (“live free or die”) (Wooley v. Maynard) c.iii) Members of the Progressive Labor Party joined the suit, claiming that Harris’s prosecution could inhibit peaceful advocacy of their agenda, and a history professor joined, claiming that prosecution made him unsure if he could teach Marxism, SC says these are not relevant because there has been no threat or arrest (Younger v. Harris) d) However, once charges are filed Younger applies, even if federal complaint was filed first and no significant proceedings have taken place d.i)No criminal proceedings were pending against the proprietor by name when the complaint was filed, but because 2 of his employees were charged and 4 copies of the movie were seized from him, he had a sufficiently substantial stake in the proceedings, but federal court had to abstain (Hicks v. Miranda) 89)Remedies available: a) Can seek prelim injunction to stay prosecution a.i) Standard for fed court issuing an injunction is abuse of discretion, isn’t subject to the restrictions of Younger because a plaintiff may challenge the constitutionality of state law in state court (Doran v. Salem Inn, Inc.) a.ii) However, Declaratory judgment has virtually the same practical impact as a formal injunction, so when granting an injunction would be inappropriate, declaratory relief should ordinarily be denied as well (Samuels v. Mackell) b) Declaratory judgment has a less intrusive effect on administering state law, and can be granted more readily b.i)federal declaratory judgment isn’t precluded if no state prosecution is pending and the federal plaintiff demonstrates genuine threat of enforcement of a disputed state criminal statute (Steffel v. Thompson) 90)Policy: a) Equity: Have always needed to show lack of an adequate remedy at law before injunctions could be issued b) Comity: For state court courts of general jurisdiction c) Federalism: Avoid needless interference of federal courts d) Separation of powers: State courts haven’t had a crack at it d.i)“On-face” challenges are not how fed courts are supposed to work d.ii) Court should be resolving concrete disputes, not “roving” through statute books and deciding 35 what’s constitutional d.iii) This is more like issuing an advisory opinion e)

Habeas Corpus

The Federal statutes:  28 USC § 2241: Power to Grant the Writ  28 USC § 2243: Issuance of writ, hearing, decision  28 USC § 2254: State custody; remedies in federal court  28 USC § 2255: Federal custody; remedies on motion attacking sentence

1996 AEDPA (28 USC § 2254): HC shall not be granted to a person in state custody as the result of a state court judgment with respect to a claim adjudicated on the merits in a state court unless the adjudication of the claim resulted in a decision that was contrary to, or involved an unreasonable application of, clearly established Federal law, as determined by the Supreme Court. Extrajudicial Detention 91)Until recently, this didn’t come up very often (used to be if INS tried to deport you) a) Now, Guantanamo Bay situations 92)Rule: If a person is held without charge, habeas corpus proceedings may be brought in federal court to test the lawfulness of the detention a) Federal and state governments rarely assert a power to restrain without criminal charge 93)O’Connor made up the standard based on Matthews v. Eldridge a) the process due is determined by weighing the private interest that will be affected by the official action against the government’s asserted interest, including the function involved and the burdens the government would face in providing greater process b) Evaluate the risk of an erroneous deprivation of the private interest if process were reduced and the probable value of additional or substitute procedural safeguards c) A citizen-detainee challenging his classification as an enemy combatant must receive notice of the factual basis for his classification and a fair opportunity to rebut the government’s assertions before a neutral decision-maker d) Evidentiary rules can be relaxed, and there can be a presumption in favor of the government’s evidence e) In Hamdi v. Rumsfeld (2004) O’Connor says that as per § 2241, a detainee may, under oath, deny facts set forth in the return or allege other material facts, take evidence by deposition, affidavit, or interrogatories, and present and rebut facts was sufficient process for quantanemo detainees held under AUMF 94)Scalia dissent in Hamdi: a) relevant question is whether there’s a special procedure for imprisoning a citizen accused of wrongdoing by aiding the enemy in wartime a.i) In such cases, citizens aiding the enemy should be tried for treason b) The President has historically secured Congress’s explicit approval of a suspension of habeas corpus b.i)The text of the Habeas Corpus Act of 1679 makes this clear c) Milligan – rejected the assertion that military assumption of jurisdiction is acceptable under the “laws and usages of war” d) Quirin – it was uncontested petitioners were members of enemy forces

36 e) Absent suspension of the writ, a citizen held when the courts are open is entitled to a criminal trial or judicial decree ordering his release e.i) The AUMF isn’t sufficiently clear to constitute suspension of the writ

The Basic Rule and Factual Issues 95)Rule: there are no Habeas petitions available for civil judgments 96)Rule: Federal Court may grant Habeas petitions when there is a federal constitutional defect in a state trial 97)Basic procedural requirements: a) The prisoner must be in custody b) The prisoner must have exhausted available state remedies b.i)If the state’s highest court has discretionary power to review decisions of lower state courts, the prisoner must apply for such review c) The prisoner must satisfy the statute of limitations c.i) AEDPA sets a 1-year limitations period for persons in custody pursuant to a state court’s judgment c.ii) Runs on the latest of 4 possible dates, the most common is (c.ii.1) the date the judgment became final by conclusion of direct review or (c.ii.2) expiration of the time for seeking habeas review d) In most cases, the prisoner must not be presenting a successive petition d.i)Successive habeas petitions not presented in prior petitions should be dismissed unless: d.ii) The claim is based on a new rule of law the Supreme Court has made retroactive to cases on collateral review; or d.iii) The claim is based on facts that couldn’t previously have been discovered and that would be sufficient to show that, but for constitutional error, no reasonable fact-finder could have found the prisoner guilty of the underlying offense 98)Adjudication of factual issues: a) Old Rule (Townson v. Sain): a.i) Evidentiary hearings (a.i.1) Basic Rule: evidentiary hearing is always available, but only mandatory in cases where the state court proceeding did not fully and fairly litigate the facts (a.i.2) In certain situations, the federal judge must hold an evidentiary hearing: (a.i.2.a) the merits of the factual dispute were not resolved in the state hearing, either at the time of the trial or in a collateral proceeding; (a.i.2.b) the state factual determination is not fairly supported by the record as a whole; (a.i.2.c) the factfinding procedure employed by the State Court was not adequate to afford a full and fair hearing; (a.i.2.d) there is a substantial allegation of newly discovered evidence; (a.i.2.e) the material facts were not adequately developed at the state court hearing; or (a.i.2.f) for any reason it appears that the state trier of fact did not afford the applicant a full and fair fact hearing. Pp. 372 U. S. 312-318. (a.i.3) In all other situations, the federal judge “has the power to receive evidence and try the facts anew” at his discretion b) New Rule (§ 1996): b.i)A federal court shouldn’t hold hearing unless: (b.i.1) Claim relies on new rule of constitutional law, OR (b.i.2) Factual predicate that could not have been previously discovered through the exercise of 37 due diligence (b.i.2.a) Test: whether petitioner “failed” to develop his/her claim (b.i.2.b) Failure means the plaintiff is at fault (b.i.2.b.i) Habeas petitioner was not at fault for juror bias, thus his Habeas petition could move forward on that ground (Michael Williams v. Taylor) 99)Standard of review: a) Rule (AEDPA): HC shall not be granted unless a decision was a.i) contrary to, or a.ii) involved an unreasonable application of, (a.ii.1) “unreasonably” does NOT mean simply wrong but means no “reasonable jurist” could reach that conclusion (Williams v. Taylor) b) clearly established Federal law, as determined by the Supreme Court b.i)must be clearly established at the time of the State Court conviction b.ii) must have been established by the Supreme Court c) Practical effect: c.i) Eliminates Teague exceptions, limits Habeas grants to the text of AEDPA 100) Policy: a) Statute requires the federal courts to do this b) HC inherent in liberty and democracy b.i)Fay v. Noia, (US 1963): res judicata doesn’t apply because the court is only acting on the body of the D, not attacking the prior judgment c) Supreme Court’s review is discretionary, don’t have time to deal with all the cases d) Takes the primary view of the fed courts protecting fed rights e) Did the SC in Williams v. Taylor intend to overrule Teague? e.i) Stevens: No. If a decision is wrong, it’s contrary to federal law (whether or not it’s really, really/ unreasonably wrong) (e.i.1) Regardless of text, this interpretation is ridiculous – why would Court uphold a ruling that is wrong? (e.i.2) Standard of review is usually de novo (no comparative expertise) (e.i.2.a) This is like Chevron deference – deference given interpretation is reasonable (e.i.2.b) There, deference promotes uniformity (e.i.2.c) Here, courts giving deference promotes disuniformity e.ii) O’Connor: Yes. What else would “unreasonable application” do in that case? Must be “contrary” and “unreasonably so”? (e.ii.1) Never want to render part of a statute superfluous

Good Bad - Detention requires justification - Adds to workload of the federal courts - Federal judges should enforce federal - Undermining the state efforts to enforce rights b/c of life tenure their own criminal laws - Protects federal rights - Inefficient - More consistent results from allowing - Distrust of state judges to apply federal courts to decide (fewer fed courts, constitution applying same law) - Frustrates states b/c of ambiguity of 14th - Extra incentive to state courts to get things amendment right - Contrary to res judicata; second chance 38 - Undermines finality - Contrary to courts’ own abstention doctrines - Criminals “go free” (though state has option of retrying the defendant) - Meritorious claims are harmed by being in flood of frivolous claims

Cognizable Claims 101) Rule: 4th amendment claims are not cognizable in Habeas petitions a) Exclusionary rule isn’t in 4th Amendment – not a personal constitutional right (meant to be preventative, not corrective) (Stone v. Powell) b) Fails the prudential balancing test of the 4th Amendment b.i)Benefit: prevents officers from conducting illegal searches b.ii) Cost: Lets guilty Ds go free - Benefits no longer outweigh the costs 102) Rule: pure “claims of innocence” are not cognizable in Habeas proceedings a) Claims of innocence cannot be the sole claim in a Habeas petition b) Exception: Court has stated that “truly persuasive” claims of innocence might be cognizable c) Exception: FC will still look at claims of clearly erroneous appellate review c.i) However, if there was any rational basis for conviction, then can’t be overturned 103) Policy: a) What is the purpose of HC proceedings? a.i) Powell: safeguard against putting innocent people in jail (a.i.1) Response: structural differences (no life tenure, elections, ect.) might not make state judges as bold as their federal counterparts a.ii) Brennan: to vindicate/safeguard constitutional rights b) Why does the Supreme Court distinguish between different types of constitutional violations for HC proceedings? b.i)Burger: doesn’t like exclusionary rule in the first place, so especially doesn’t like it in HC b.ii) Powell: state courts are just as capable of deciding federal questions as federal courts b.iii) Brennan: HC statute doesn’t distinguish between types of constitutional violations (b.iii.1) Fears that this will be a slippery slope – that HC will be lost for other claims as well – Miranda violations, double jeopardy, etc. (not guilt-related claims) (b.iii.2) Not based on person’s factual innocence (serve to vindicate other federal rights that don’t go to substantive charge of the crime) (b.iii.2.a) Claim of being tried before for the same defense has nothing to do with your guilt before (b.iii.3) Majority does suggest that this is all about guilt v. non-guilt related: (b.iii.3.a) “Innocent man”; “convicted defendant is usually not related to basic justice of incarceration” c) why no claims of innocence? c.i) There’s always executive clemency

39 New Rules and Standard of Review 1) Rule: An individual may not seek to enforce a new rule of law in federal habeas corpus proceedings if the new rule was announced after the petitioner’s conviction became final a) Additionally, Habeas petitioners may not seek to establish a wholly new rule or to apply a settled precedent in a novel way that would result in the creation of a new rule. 2) Exceptions: new rules may be applied retroactively to final convictions IF the rule… a) places “certain kinds of primary, private individual conduct beyond the power of the criminal law-making authority to proscribe.” a.i) I.e., sodomy a.ii) Batson was more about procedure, whereas this is about whether what you did could be criminalized at all b) requires the observance of “those procedures that... are implicit in the concept of ordered liberty.’” b.i)Has to be a fundamental, watershed new rule about the accuracy of your conviction b.ii) Must be truth-related 3) Practical effect: a) Procedural consequence: If a rule won’t apply retroactively to all persons similarly situated (Habeas petitioners), FC won’t even look at it 4) Policy: a) Is the court’s method of analysis here backwards? a.i) Stevens: courts do this all the time with harmless error – first decide whether there’s an error in the criminal case, and then decide whether or not it’s harmless (a.i.1) Strictly following the logic in Teague, seems you should first figure out if there was harmless error, and then decide whether it was truly error a.ii) Brennan: Court should decide merits of the claim, then later figure out whether it applies retroactively, and should be decided in favor of Teague (a.ii.1) Even if others similarly situated wouldn’t get the benefit, should still give as much justice as possible to Teague

Procedurally Defaulted Claims 5) Old Old Rule (Daniels v. Allen): a) Federal Courts cannot reverse a state court decision when there was an adequate and independent state law ground a.i) Court can’t rule on federal issue where there is a mixed claim because it wouldn’t change the judgment (for one which is also supportable on the state issue) a.ii) would just be an advisory opinion 6) New Old Rule (Fay v. Noia): a) A prisoner’s “default” under state procedure does not bar a Habeas petition unless the prisoner deliberately bypassed the appeal to play the system a.i) Even here, judge can use his discretion not to bar the petition b) Policy: This isn’t an appeal – just ordering state court to release the prisoner 7) Current Rule (Wainwright v. Sykes): a) Where there was a waiver at trial, HC should be barred unless P can show: a.i) Cause for failing to object AND (a.i.1) Brady violation (prosecutor not turning over exculpatory evidence) is typically good cause (a.i.2) Screw-ups by counsel are typically not good cause (a.i.2.a) Can still make ineffective assistance of counsel claims 40 (a.i.3) Relying upon existing law is good cause (a.i.3.a) If the rule you were relying upon didn’t exist at the time, you have good cause a.ii) Prejudice to the decision b) Exception: actual innocence b.i)Not by itself a basis for HC, but if there’s evidence of actual innocence, this excuses you from meeting the Cause and Prejudice standard b.ii) Defined very narrowly c) Policy: c.i) Respect for the contemporaneous objection rule c.ii) Fay v. Noia encourages sandbagging (c.ii.1) If you don’t bring it up, you could be acquitted – if you get convicted, you can then bring it up later and get it washed out c.iii) If courts didn’t respect COR, state courts might be less likely to enforce it c.iv) Leads to state trial being the “main event” c.v) Brennan: Real reason for failure to raise these is that attorneys are ignorant and incompetent, ridiculous to assume that there’s sandbagging (c.v.1) Better to take your best shot in state court and get HC review – HC review can be a lot less favorable than raising your claim at the right time (c.v.2) Shouldn’t discipline client for lawyer’s flaws 8) Statute of limitations a) One year after a.i) Cert denied by SC a.ii) Or decide not to seek cert. b) Only have to make it through the state appeals process for your decision to be final b.i)

41